Nervous System Pathology A Case-based Learning Approach Prepared and presented by Marc Imhotep Cray, M.D.
“The mediocre teacher tells. The good teacher explains. The superior teacher demonstrates. The great teacher inspires.” William Arthur Ward 1
False-color confocal micrograph of a section through the brain, showing an individual neuron of the cerebellum with extensive processes arising from a cell body. From: Widmaier, EP; Raff, H; Strang, KT. Vander’s Human Physiology: The Mechanisms of Body Function 14th Ed. New York, NY: McGraw-Hill Education, 2016. Pg. 136.
Learning Outcomes By the end of this series the learner should be able to: 1. Describe the pathological characteristics of epidural, subdural and subarachnoid hemorrhages 2. Describe CNS aneurysms. List their types, causes and effects 3. Describe the morphology and pathogenesis of brain infarction. 4. Give a list of demyelination diseases of the nervous system 5. Describe the etiology and pathological findings of multiple sclerosis 6. List neurodegenerative diseases of the nervous system and explain their pathogenesis 7. Describe the pathogenesis and pathological findings of Alzheimer's disease Marc Imhotep Cray MD
2
Learning Outcomes cont’ed. By the end of this series the learner should be able to: 8. Classify CNS & PNS neoplasms 9. Describe the morphological features of the common intracranial neoplasms 10. Describe the pathological effects of CNS & PNS tumors 11. Discuss the etiology of peripheral neuropathy 12. List the causes and effects of intracranial space occupying lesions 13. Discuss the pathology of hydrocephalus 14. Describe the pathology of brain abscess 15. Describe the pathological features of encephalitides
Marc Imhotep Cray MD
3
Classes of Nervous System Disorders
Marc Imhotep CrayCI. MD Baron SJ and Lee Lange Pathology Flash Cards. New York: McGraw-Hill, 2009..
4
Topical Outline
Introduction/Neuroanatomy Review Congenital Diseases Tuberous Sclerosis Multiple Sclerosis Guillain-Barre Syndrome Alzheimer Disease Huntington Disease Parkinson Disease Amyotrophic Lateral Sclerosis Ischemic Stroke Hemorrhagic Stroke Berry Aneurysm and Subarachnoid Hemorrhage Epidural and Subdural Hematoma
Marc Imhotep Cray MD
Hydrocephalus Syringomyelia Myasthenia Gravis Seizures Pyogenic and Viral Meningitis Retinoblastoma Glioblastoma Multiforme Meningioma Oligodendroglioma Schwannoma Medulloblastoma Ependymoma Neuroblastoma Neurofibromatosis Type 1 5
Introduction / Neuroanatomy Review
Organization of Nervous System Normal Brain, Gross and Microscopic Functional Areas of Brain Blood Supply to Cerebrum Meninges Cerebrospinal Fluid Ventricular System Blood Brain Barrier
For a Comprehensive Neuroscience Video Edu. Review see: 2-Minute Neuroscience_Neuroscientifically Challenged Marc Imhotep Cray MD
6
Organization of Nervous System CENTRAL NERVOUS SYSTEM (CNS)
BRAIN & SPINAL CORD
AFFERENT
EFFERENT
(Sensory)
(Motor)
NERVES
NERVES
EXTEROCEPTORS
EFFECTOR ORGANS
Marc Imhotep Cray MD
INTEROCEPTORS
SKELETAL MUSCLES
VOLUNTARY Monosynaptic
PERIPHERAL NERVOUS SYSTEM (PNS)
SOMATIC
AUTONOMIC
SMOOTH MUSCLE, CARDIAC MUSCLES AND GLANDS
INVOLUNTARY Pre & Post Ganglionic Fiber
7
Divisions of Nervous System
From: F. Fay Evans-Martin, Ph.D. Introduction by Denton A. Cooley, M.D. The Nervous System (Your Body How It Works) Marc Imhotep Cray MD
8
Normal Brain, Gross and Microscopic Follow
Neuroanatomy Recall: Grey matter (outside layer in brain) contains numerous cell bodies and relatively few myelinated axons; White matter (outside layer in spinal cord) contains relatively few cell bodies and is composed chiefly of long-range myelinated axons Color difference arises mainly from whiteness of myelin. Learn more: https://human-memory.net/gray-white-matter/ Marc Imhotep Cray MD
9
Normal brain, gross
Klatt EC. Robbins and Cotran Atlas of Pathology, 3rd Ed. 2015.
Marc Imhotep Cray MD
10
Normal brain, gross
Klatt EC. Robbins and Cotran Atlas of Pathology, 3rd Ed. 2015.
Marc Imhotep Cray MD
11
Normal brain, gross
Klatt EC. Robbins and Cotran Atlas of Pathology, 3rd Ed. 2015.
Marc Imhotep Cray MD
12
Normal brain, gross
Klatt EC. Robbins and Cotran Atlas of Pathology, 3rd Ed. 2015.
Marc Imhotep Cray MD
13
Normal brain, gross
Klatt EC. Robbins and Cotran Atlas of Pathology, 3rd Ed. 2015.
Marc Imhotep Cray MD
14
Normal brain, gross
Klatt EC. Robbins and Cotran Atlas of Pathology, 3rd Ed. 2015.
Marc Imhotep Cray MD
15
Normal neocortex, microscopic
Klatt EC. Robbins and Cotran Atlas of Pathology, 3rd Ed. 2015.
Marc Imhotep Cray MD
16
Normal hippocampus, microscopic
Klatt EC. Robbins and Cotran Atlas of Pathology, 3rd Ed. 2015.
Marc Imhotep Cray MD
17
Normal cerebellum, microscopic
Klatt EC. Robbins and Cotran Atlas of Pathology, 3rd Ed. 2015.
Marc Imhotep Cray MD
18
Normal brain, microscopic
Klatt EC. Robbins and Cotran Atlas of Pathology, 3rd Ed. 2015.
Marc Imhotep Cray MD
19
Functional Areas of Brain Lobe
Functional Area
Frontal
Motor and premotor cortex, frontal eye fields, Broca speech area, executive functioning (concentration, judgment, and problem solving)
Temporal Primary auditory cortex, memory, Wernicke area Parietal
Sensory cortex, spatial orientation
Occipital
Primary visual cortex
Marc Imhotep Cray MD
20
Lobes of Brain (Medial view of a halved human brain) )
Marc Imhotep Cray MD
21
Motor and Sensory Regions of Cerebrum
Marc Imhotep Cray MD
22
Blood Supply to Cerebrum Artery
Origin and Region Supplied
Anterior cerebral artery Middle cerebral artery Posterior cerebral
Branches from internal carotid; supplies medial surface of brain, anterior limb of internal capsule, basal ganglia, and frontal pole Branches from internal carotid; supplies lateral surface of brain, posterior limb of internal capsule, and basal ganglia artery Branches from basilar artery; supplies occipital pole, inferomedial temporal lobes, and the thalamus
Lateral striate arteries
Branches from middle cerebral artery; supplies internal capsule and basal ganglia
Marc Imhotep Cray MD
23
Marc Imhotep Cray MD
24
Organization of human cerebral circulation The brain receives its blood supply through the right and left internal carotid as well as a pair of vertebral arteries. The arteries ultimately join the Circle of Willis, an anatomical anastomosis. Different arteries that branch off the Circle of Willis to distribute blood supply to the two cerebral hemispheres are shown (University of Miami Health System http://surgery.med.miami.edu/ima ges/Circulation_of_ brain.gif)
Marc Imhotep Cray MD
See Video: Anatomy_Brain (Circle of Willis and Stroke)
25
Circle of Willis Anterior and posterior circulations meet at Circle of Willis, which rests at top of brainstem
Structure Circle of Willis is a part of cerebral circulation and is composed of following arteries: Anterior cerebral artery (left and right) Anterior communicating artery Internal carotid artery (left and right) Posterior cerebral artery (left and right) Posterior communicating artery (left and right) Function Arrangement of brain's arteries into circle of Willis creates redundancy for collateral circulation in cerebral circulation If one part of circle becomes blocked or narrowed, blood flow from other blood vessels can often preserve cerebral perfusion well enough to avoid symptoms of ischemia Marc Imhotep Cray MD See Video Mini-Lecture: Circle of Willis 3D Anatomy Tutorial
26
Circle of Willis (2)
Marc Imhotep Cray MD
27
Meninges Diagram of section of top of brain showing meninges and subarachnoid space
Marc Imhotep Cray MD
28
Cerebrospinal fluid circulation Cerebrospinal fluid circulates in subarachnoid space around brain and spinal cord, and in ventricles of brain
Cerebrospinal fluid has three main functions: CSF protects brain and spinal cord from trauma CSF supplies nutrients to nervous system tissue CSF removes waste products from cerebral metabolism Marc Imhotep Cray MD
29
CSF circulates in spaces around & within brain
Marc Imhotep Cray MD
30
Distribution of CSF
Marc Imhotep Cray MD
31
4 vials of human cerebral spinal fluid of normal appearance, collected via lumbar puncture from the L3/L4 disk space.
Marc Imhotep Cray MD
32
Ventricular System A set of four interconnected cavities (ventricles or lumen) in brain Within each ventricle is a region of choroid plexus where CSF is produced Continuous w central canal of spinal cord from fourth ventricle, allowing for flow of CSF to circulate
Marc Imhotep Cray MD
33
Ventricular System (2)
System comprises four ventricles: Lateral ventricles right and left (one for each hemisphere) Third ventricle Fourth ventricle Marc Imhotep Cray MD
34
Size and location of ventricular system in human head_3D Animation
MRI showing flow of CSF
Images from a patient w normal pressure hydrocephalus (NPR) showing pulsations of CSF w heartbeat.
Online Version Marc Imhotep Cray MD
35
2-Minute Neuroscience: The Ventricles
Online Version Marc Imhotep Cray MD
36
Blood Brain Barrier Blood Brain Barrier, Animation
Online Version
Marc Imhotep Cray MD
37
Cerebral Hemispheres_ 3D Animation
Online Version Marc Imhotep Cray MD
38
Congenital Diseases
Marc Imhotep Cray MD
39
Vignette 1 A baby boy is born via an uncomplicated vaginal delivery. You notice a large birthmark, associated with dimples and hairy tufts, at the base of his back. Upon questioning his mother, you learn that she did not take any prenatal vitamins or receive prenatal care during her pregnancy. Although the child initially appears normal, you suggest that the child be evaluated for a disorder caused by a neural tube defect and you fear that he may develop autonomic and motor deficits later in life. What is the Diagnosis?
Marc Imhotep Cray MD
40
Congenital CNS Abnormalities Neural Tube Defects Etiology: Associated with folate deficiency during initial gestation and elevated α-fetoprotein Spina bifida: Failure of posterior end of neural tube to close; results in vertebral bony defect through which meninges can herniate (meningocele) or meninges and spinal cord can herniate (meningomyelocele) resulting in neurologic symptoms; bony defect may also be asymptomatic until later in life, when neurologic symptoms appear (spina bifida occulta) Encephalocele: Defect in cranium allows for out-pouching of brain through skull
Anencephaly: Failure of anterior end of neural tube to close; results in absence of fetal brain and often absence of overlying skull
Marc Imhotep Cray MD
41
Congenital CNS Abnormalities (2) Holoprosencephaly Pathology: Failure of embryo’s forebrain to divide into bilateral cerebral hemispheres, leading to incomplete separation of cerebral hemispheres Clinical Manifestations: Facial and neurological defects
Arnold-Chiari Malformation
Pathology: Characterized by a small posterior fossa, resulting in displacement of cerebellum and medulla through foramen magnum Clinical manifestations: Hydrocephalus; associated strongly with thoracolumbar meningomyelocele and syringomyelia
Dandy-Walker Malformation Pathology: Characterized by large posterior fossa with replacement of cerebellar vermis with large cyst; associated with brainstem nuclei dysplasias Clinical Manifestations: Seizures and cerebellar dysfunction Marc Imhotep Cray MD
42
Neural tube defects
First Aid for the USMLE Step1, 2020, Pg. 491.
Neuropores fail to fuse (4th week) persistent connection between amniotic cavity and spinal canal Associated with maternal diabetes and folate deficiency Inc. α-fetoprotein (AFP) in amniotic fluid and maternal serum (except spina bifida occulta = normal AFP) Inc. acetylcholinesterase (AChE) in amniotic fluid is a helpful confirmatory test Marc Imhotep Cray MD
43
Posterior fossa malformations Chiari I malformation Ectopia of cerebellar tonsils inferior to foramen magnum (1 structure) Congenital, usually asymptomatic in childhood, manifests in adulthood w headaches and cerebellar symptoms Associated w spinal cavitations (eg, syringomyelia) Chiari II malformation Herniation of cerebellar vermis and tonsils (2 structures) through foramen magnum w aqueductal stenosis noncommunicating hydrocephalus Usually assoc. w lumbosacral myelomeningocele (may present as paralysis/sensory loss at and below level of lesion) o More severe than Chiari I, usually presents early in life Marc Imhotep Cray MD
44
Vignette 2 A 25-year-old woman presents to your office complaining of diminished sensation in both arms. Further neurologic evaluation of her arms reveals that she has diminished pain and temperature sensation, but that her touch sensation and proprioception are intact. She demonstrates no other neurologic deficits. When a MRI reveals cystic dilation in the center of the cervical spinal cord, you refer her to a neurosurgeon for treatment of her condition. What is the Diagnosis?
Marc Imhotep Cray MD
45
Syringomyelia Etiology: Often assoc. w Arnold-Chiari malformation; also caused by intraspinal neoplasms or trauma Pathology: Spinal cord: Formation of fluid-filled cavity often extending from central canal usually in cervical region of cord (most commonly C7-T1) results in destruction of adjacent gray and white matter (crossing fibers of spinothalamic tract) with resultant reactive gliosis Clinical Manifestations: Loss of pain and temperature sensation of upper extremities usually w preservation of touch and proprioception; may eventually progress to involve motor and other sensory tracts Treatment: Surgical drainage of cavity
Marc Imhotep Cray MD
46
Posterior fossa malformations (2) Chiari I malformation
Dandy-Walker Malformation
First Aid for the USMLE Step1, 2020, Pg. 491.
Dandy-Walker malformation Agenesis of cerebellar vermis cystic enlargement of 4th ventricle (arrow in B ) that fills enlarged posterior fossa Assoc. w noncommunicating hydrocephalus, spina bifida Note: A syrinx is a fluid-filled (cystic) cavity within the spinal cord (syringomyelia) Marc Imhotep Cray MD or brain stem (syringobulbia).
47
Syringomyelia Cystic cavity (syrinx) within central canal of spinal cord (yellow arrows in A ); Fibers crossing in anterior white commissure (spinothalamic tract) are typically damaged first Associated with Chiari I malformation (red arrow in A shows low-lying cerebellar tonsils), scoliosis and other congenital malformations; acquired causes include trauma and tumors Most common location cervical > thoracic >> lumbar First Aid for the USMLE Step1, 2020, Pg. 492. ( Syrinx = tube, as in “syringe.”) Marc Imhotep Cray MD
48
Syringomyelia (2) Results in a “capelike,” bilateral, symmetrical loss of pain and temperature sensation in upper extremities (fine touch sensation is preserved)
First Aid for the USMLE Step 1, 2020, Pg. 492. Marc Imhotep Cray MD
49
Hydromyelia, gross, compared with syringomyelia, microscopic
Klatt EC. Robbins and Cotran Atlas of Pathology, 3rd Ed. 2015.
Marc Imhotep Cray MD
50
Anencephaly, gross
Klatt EC. Robbins and Cotran Atlas of Pathology, 3rd Ed. 2015.
Marc Imhotep Cray MD
51
Meningomyelocele, gross
Klatt EC. Robbins and Cotran Atlas of Pathology, 3rd Ed. 2015.
Marc Imhotep Cray MD
52
Encephalocele, gross
Klatt EC. Robbins and Cotran Atlas of Pathology, 3rd Ed. 2015.
Marc Imhotep Cray MD
53
Rachischisis, gross
Klatt EC. Robbins and Cotran Atlas of Pathology, 3rd Ed. 2015.
Marc Imhotep Cray MD
54
Exencephaly, gross
Klatt EC. Robbins and Cotran Atlas of Pathology, 3rd Ed. 2015.
Marc Imhotep Cray MD
55
Arnold-Chiari I malformation, gross
Klatt EC. Robbins and Cotran Atlas of Pathology, 3rd Ed. 2015.
Marc Imhotep Cray MD
56
Arnold-Chiari I malformation, MRI
Klatt EC. Robbins and Cotran Atlas of Pathology, 3rd Ed. 2015.
Marc Imhotep Cray MD
57
Arnold-Chiari II malformation, gross
Klatt EC. Robbins and Cotran Atlas of Pathology, 3rd Ed. 2015.
Marc Imhotep Cray MD
58
Dandy-Walker malformation, gross
Klatt EC. Robbins and Cotran Atlas of Pathology, 3rd Ed. 2015.
Marc Imhotep Cray MD
59
Lissencephaly Failure of neuronal migration resulting in a “smooth brain” that lacks sulci and gyri May be associated with microcephaly, ventriculomegaly
Klatt EC. Robbins and Cotran Atlas of Pathology, 3rd Ed. 2015.
Marc Imhotep Cray MD
60
Alobar holoprosencephaly, gross Failure of embryonic forebrain (prosencephalon) to separate into 2 hemispheres; Occurs during weeks 5-6; May be related to mutations in sonic hedgehog signaling pathway; Assoc. w other midline defects including cleft lip/palate (moderate form) and cyclopia (severe form) ; inc. risk for pituitary dysfunction (eg, diabetes insipidus); Can be seen with Patau syndrome (trisomy 13)
Klatt EC. Robbins and Cotran Atlas of Pathology, 3rd Ed. 2015. Marc Imhotep Cray MD
61
Holoprosencephaly
Klatt EC. Robbins and Cotran Atlas of Pathology, 3rd Ed. 2015. Marc Imhotep Cray MD
62
Vignette 3 A 6-year-old boy presents to your office complaining of red nodules that have appeared on his face. He is mentally retarded and has a seizure disorder. His mother suffers from similar symptoms and has recently been diagnosed with a rhabdomyoma of the heart and renal angiomyolipomas. You begin to suspect that this boy is suffering from an autosomal dominant genetic disorder. What is the Diagnosis?
Marc Imhotep Cray MD
63
Tuberous Sclerosis Etiology: Autosomal dominant disorder resulting from a mutation in one of several different genes Tumor suppressor genes TSC1 on chromosome 9 (hamartin), TSC2 on chromosome 16 (tuberin)
Pathology: Brain hamartoma (cortical tuber): Firm nodule located in cerebral cortex composed of disorganized array of neurons with large vesicular nuclei and eosinophilic cytoplasm Also assoc. w neoplasms occurring outside CNS, including cardiac rhabdomyomas, adenoma sebaceum of face (lesion consisting of malformed blood vessels), renal angiomyolipomas (lesion consisting of malformed blood vessels, adipocytes, and smooth muscle), and cysts of bone and lung Marc Imhotep Cray MD
64
Tuberous Sclerosis (2) Clinical Manifestations: Seizures and mental retardation in infancy; red nodules on face (adenoma sebaceum), which appear betw. ages of 5 and 10; symptoms related to cardiac rhabdomyoma and renal angiomyolipoma Treatment: Symptomatic (control seizures); genetic counseling
Patient with facial angiofibromas caused by tuberous sclerosis Marc Imhotep Cray MD
CT showing multiple angiomyolipomas of kidney in 65 a patient with suspected TSC
Tuberous sclerosis, gross
Klatt EC. Robbins and Cotran Atlas of Pathology, 3rd Ed. 2015.
Marc Imhotep Cray MD
66
Tuberous sclerosis
First Aid for the USMLE Step1, 2020, Pg. 525.
HAMARTOMASS Hamartomas in CNS and skin, Angiofibromas (C), Mitral regurgitation, Ash-leaf spots (D), cardiac Rhabdomyoma, (Tuberous sclerosis), autosomal dOminant; Mental retardation (intellectual disability), renal Angiomyolipoma (E) , Seizures, Shagreen patches. Marc Imhotep Cray MD
67
Check Up A 1-year-old girl develops seizures, ash-leaf pigmented lesions on the trunk, sebaceous adenomas, and a shagreen patch (fleshcolored soft plaque) on her lumbosacral region. What is the diagnosis? A 40-year-old woman complains of occipital headaches, weakness/numbness in her hands and feet, and has downbeat nystagmus on physical examination. An MRI shows tonsillar herniation below the foramen magnum. What is the diagnosis?
Marc Imhotep Cray MD
68
Degenerative Disorders
Marc Imhotep Cray MD
69
CNS Neurodegenerative diseases Neurodegenerative diseases of CNS include: Parkinson’s disease Huntington disease Alzheimer’s disease Multiple Sclerosis (MS) Amyotrophic Lateral Sclerosis (ALS) These devastating illnesses are characterized by progressive loss of selected neurons in discrete brain areas, resulting in characteristic disorders of movement, cognition, or both
Marc Imhotep Cray MD
70
Basal Nuclei (Ganglia) Components Caudate nucleus Putamen Globus pallidus Grouping of the Basal Nuclei (Ganglia) The striatum consists of caudate nucleus and putamen The lentiform nucleus consists of globus pallidus and putamen The corpus striatum consists of lentiform nucleus and caudate nucleus NB: Basal Nuclei The term basal ganglia is a misnomer. The cells forming these structures are not “ganglia”—a term reserved to describe aggregations of neuronal cell bodies[somata] (groups of nerve cell bodies ) in the peripheral nervous system—but “nuclei” in the central nervous system. Marc Imhotep Cray MD
71
Vignette 4 A 78-year-old woman is brought to your clinic by her son and daughter. They tell you that she has been very forgetful lately and has twice wandered out of her house and gotten lost, requiring the police to bring her back. Upon speaking with the woman, you note that her short-term memory is compromised and that she has trouble finding the words to express what she wants to say. An MRI of the brain does not reveal any evidence of a stroke. You suspect that a biopsy of this woman’s brain would reveal neuritic plaques and neurofibrillary tangles. What is the Diagnosis?
Marc Imhotep Cray MD
72
Dementia Dementia is a progressive global decline in intellectual capacity that occurs with increasing frequency with advancing age Two most commonly encountered forms are Alzheimer’s disease (AD) (sometimes familial) and Vascular (multi-infarct) dementia (VaD) Less common dementias are Huntington’s disease (an inherited condition) and Pick’s disease Marc Imhotep Cray MD
73
Alzheimer disease Etiology and Epidemiology: Etiology unknown, but theories involve abnormal expression of amyloid gene resulting in increased amyloid beta (Aβ protein), deficiency of choline acetyltransferase leading to decreased acetylcholine levels, or atrophy of nucleus basalis of Meynert Familial Alzheimer disease involves o mutations in amyloid precursor protein (APP) gene on chr 21, o mutations in presenilin genes (chr 1,14), and o Ɛ 4 allele of apolipoprotein E (chr 19) Affects 50% of people > 85 years old NB: Lewy body dementia, also known as dementia w Lewy bodies, is second most common type of progressive dementia after AD dementia. Protein deposits, called Lewy bodies, develop in nerve cells in brain regions involved in thinking, memory and movement (motor control). Marc Imhotep Cray MD
74
Alzheimer disease (2) Pathology Gross: Cortical atrophy of brain with widening of sulci and ventricles Microscopic: Neurofibrillary tangles composed of tau protein within cytoplasm that displace nucleus; neuritic plaques (spherical cluster with Aβ protein core and peripheral astrocytes); amyloid angiopathy; Hirano bodies (eosinophilic bodies in hippocampal cells); granulovacuolar degeneration (cytoplasmic vacuoles in hippocampal cells) NB: The hippocampus is area of brain demonstrating greatest degree of atrophy in Alzheimer's disease Hippocampal atrophy on MRI is highly suggestive of the diagnosis Marc Imhotep Cray MD
75
Alzheimer Disease Pathology
Marc Imhotep Cray MD
Buja, LM; Krueger GR. Netter’s Illustrated Human Pathology 2nd Ed. Illustrations by Frank H. Netter, MD. Philadelphia, PA: SaundersElsevier, 2014; Pg. 482, Fig. 13-38.
76
Alzheimer disease (3) Clinical Manifestations: Dementia presenting with progressive memory disturbances, disorientation, aphasias, visuospatial deficits, loss of motor skills or incontinence Initial symptoms: Impairments involving recent memory
Treatment and Prognosis Donepezil (acetylcholinesterase inhibitor) to slow progression Progressive disease with no cure Note: Pick disease also causes dementia, but tends to affect women more than men Histopathologically, characterized by cortical atrophy of frontotemporal lobes and Pick bodies (cytoplasmic inclusion bodies made of neurofilaments) Initial symptoms: personality & behavioral changes (apathy, socially Marc Imhotep Cray MD inappropriate behavior)
77
Alzheimer disease (4) Most common cause of dementia in elderly Down syndrome patients have inc. risk of developing Alzheimer disease, as APP is located on chromosome 21; dec. ACh.
Associated with following altered proteins: ApoE-2: dec. risk of sporadic form ApoE-4: inc. risk of sporadic form APP, presenilin-1, presenilin-2: familial forms (10%) with earlier onset
Widespread cortical atrophy (normal cortex B ; cortex in Alzheimer disease C ), especially hippocampus (arrows in B and C ). Narrowing of gyri and widening of sulci. Senile plaques D in gray matter: extracellular β-amyloid core; may cause amyloid angiopathy intracranial hemorrhage; Aβ (amyloid-β) synthesized by cleaving amyloid precursor protein (APP). Neurofibrillary tangles E : intracellular, hyperphosphorylated tau protein = insoluble cytoskeletal elements; number of tangles correlates with degree of dementia. Hirano bodies—intracellular eosinophilic proteinaceous rods in hippocampus Adapted from: Le T ; Bhushan V. First Aid for the USMLE Step 1 2020. McGraw-Hill, 2020, Pg. 520-21.
Marc Imhotep Cray MD
78
Alzheimer disease, gross
Klatt EC. Robbins and Cotran Atlas of Pathology, 3rd Ed. 2015.
Marc Imhotep Cray MD
79
Alzheimer disease, microscopic
Klatt EC. Robbins and Cotran Atlas of Pathology, 3rd Ed. 2015.
Marc Imhotep Cray MD
80
Alzheimer disease, microscopic
Klatt EC. Robbins and Cotran Atlas of Pathology, 3rd Ed. 2015.
Marc Imhotep Cray MD
81
Pick disease, gross Frontotemporal dementia: Formerly called Pick disease Early changes in personality and behavior (behavioral variant), or aphasia (primary progressive aphasia) May have associated movement disorders
Klatt EC. Robbins and Cotran Atlas of Pathology, 3rd Ed. 2015. Marc Imhotep Cray MD
82
Q&A Check-Up A 54-year-old man is brought to the physician by his daughter. She says that her father has been acting strangely over the past 2 years. He makes inappropriate sexual jokes, has little regard for social rules, is often irritable, and is borderline aggressive at times. The patient denies that his personality is any different from normal. When speaking with the patient, the physician notices that he has minimal verbal output and repeats "it is what it is" when asked questions about his strange behavior. On physical examination, he appears unkempt but otherwise has no significant findings. This patient most likely has a condition involving which of the following? A. Caudate nucleus B. Frontal cortex C. Hippocampus D. Parietal cortex E. Substantia nigra F. Subthalamic nucleus
Marc Imhotep Cray MD
83
Answer Important structures in the frontal lobe include the motor cortex, Broca's speech area, frontal eye fields, and the prefrontal cortex. Lesions involving the prefrontal cortex cause inappropriate behavior, impaired judgment, and poor problem-solving skills (executive dysfunction). Incontinence and gait disturbances may also be seen. Frontotemporal dementia (Pick's disease) is characterized by degeneration of the frontal lobes that eventually progresses to include the temporal lobes. Characteristic symptoms include early personality and behavioral changes (eg, disinhibition, apathy, social inappropriateness, compulsive behaviors) and altered speech patterns (eg, paucity of speech, repeated phrases). Neurocognitive deficits occur later in the course of the illness. When evaluating a patient with dementia, frontotemporal dementia should be differentiated from the more common Alzheimer disease, which presents initially with impairment involving recent memory.
Source: UWorld Step 1, 2015. Neurology/Pathology Q#40 Marc Imhotep Cray MD
84
Comparison of frontotemporal dementia & AD
Marc Imhotep Cray MD
85
Vignette 5 A 48-year-old man presents to your clinic complaining of involuntary movements of his arms and legs. He tells you that his mother had similar symptoms, which eventually progressed to dementia. Physical examination and history reveal involuntary jerky movements, flattened affect, and poor concentration. When an MRI of the brain demonstrates atrophy of the caudate nucleus and putamen as well as dilatation of the ventricles, you fear that this patient will eventually succumb to the same dementia as his mother. What is the Diagnosis?
Marc Imhotep Cray MD
86
Huntington Disease Etiology: Autosomal dominant disorder associated with increased number of CAG repeats in Huntington disease gene on chr 4 Pathology Gross: Atrophy of caudate nucleus and putamen; may also see atrophy of globus pallidus and frontal lobe; dilation of lateral and third ventricles Microscopic: Loss of striatal neurons (GABAergic neurons); fibrillary gliosis
Marc Imhotep Cray MD
87
Huntington Disease (2) Clinical Manifestations: Progressive disorder that initially manifests betw. ages of 40 and 50; chorea (involuntary jerky movements); cognitive impairment; mood disturbances Eventually progresses to severe dementia
Treatment and Prognosis: Symptomatic treatment for dyskinesia and mood disturbances Usually fatal within 15–20 years of diagnosis Note: HD, as well as fragile X syndrome and myotonic dystrophy, demonstrates anticipation a phenomenon in which number of repeats increases w each generation and results in more severe disease manifestations
Marc Imhotep Cray MD
88
Chorea is a term applied to rapid, complex, and varied movements of body, especially distal limbs DDx includes Sydenham chorea (ARF), SLE, chorea gravidarum (in pregnant women), drug effects, and Huntington chorea o Abnormal facial & limb movements, behavioral disturbances, and progressive dementia characterize HD o HD is a degenerative disorder w an autosomal dominant inheritance pattern, w onset usually after age of 40 years o Genetic mutation is carried by approx. 50% of offspring o Autopsy reveals severe shrinkage of caudate nucleus & cortical atrophy, especially of frontal lobes Marc Imhotep Cray MD
Buja, LM; Krueger GR. Netter’s Illustrated Human Pathology 2nd Ed. Illustrations by Frank H. Netter, MD. Philadelphia, PA: Saunders- 89 Elsevier, 2014; Pg. 484, Fig. 13-40.
Huntington disease, gross
Klatt EC. Robbins and Cotran Atlas of Pathology, 3rd Ed. 2015.
Marc Imhotep Cray MD
90
Vignette 6 A 64-year-old man presents to your neurology clinic complaining of unsteadiness. As you obtain a history from this patient, you notice that he has expressionless facies and a pill-rolling tremor at rest. Physical examination reveals a shuffling gait, rigidity in response to passive movement, and bradykinesia. You suspect that the neurons of his substantia nigra may contain Lewy bodies and you prescribe levodopa to treat his symptoms. What is the Diagnosis?
Marc Imhotep Cray MD
91
Parkinson Disease Etiology and Epidemiology: Etiology unknown; Usually presents in people over age of 50 Pathology and Pathophysiology Gross: Pale substantia nigra and locus ceruleus Microscopic: Loss of pigmented dopaminergic neurons in substantia nigra with gliosis; Lewy bodies (eosinophilic, intracytoplasmic inclusion bodies) in substantia nigra neurons Pathophysiology: Loss of dopaminergic input to striatum results in loss of stimulation of basal ganglia motor circuit NB: Lewy body dementia, also known as dementia w Lewy bodies, is second most common type of progressive dementia after AD dementia. Protein deposits, called Lewy bodies, develop in nerve cells in brain regions involved in thinking, memory and movement (motor control). Marc Imhotep Cray MD
92
Neuropathology of Parkinson Disease:  In addition to an abundance of inhibitory dopaminergic neurons, neostriatum is also rich in excitatory cholinergic neurons that oppose action of dopamine  Many of symptoms of parkinsonism reflect an imbalance between excitatory cholinergic neurons and diminished number of inhibitory dopaminergic neurons
Raff RB, Rawls SM, Beyzarov EP. Netter's Illustrated Pharmacology, Updated Edition. Philadelphia: Sanders, 2014. Marc Imhotep Cray MD
93
Parkinson Disease Clinical Manifestations: Symptom constellation of pillrolling tremor, bradykinesia, shuffling gait, rigidity, postural instability, and expressionless facies (all together known as parkinsonism) Of patients with Parkinson disease, 10%–15% develop dementia Treatment Pharmacologic therapy (amantadine, anticholinergics, levodopa, dopamine agonists, MAO-B inhibitors) Note: Other causes for parkinsonism include repeated trauma (as w boxers), drugs (especially MPTP), postencephalitic parkinsonism (observed after influenza pandemic in early 1900s), and Shy-Drager syndrome (parkinsonism w orthostatic hypotension and autonomic dysfunction) Marc Imhotep Cray MD
94
Parkinsonism: Symptoms and Defect Parkinsonism is a progressive neurodegenerative disease that adversely affects motor neuron control: Major early symptoms are: tremor at rest; bradykinesia ;muscle rigidity (cogwheel) and flat facial affect If untreated, condition worsens, leading eventually to complete immobility and early mortality Prevalence is approximately 2% in persons older than 65 years A genetic predisposition likely, but environmental factors (including viral infections and neurotoxins) may play a role Marc Imhotep Cray MD
95
Clinical Signs of Parkinson’s Disease:
Raff RB, Rawls SM, Beyzarov EP. Netter's Illustrated Pharmacology, Updated Edition. Philadelphia: Sanders, 2014. Marc Imhotep Cray MD
96
Parkinsonism: Symptoms and Defect (2) Most distinctive neuropathologic finding is progressive loss of dopaminergic neurons of pars compacta of substantia nigra
Projections of dopaminergic neurons from substantia nigra correlate with motor and cognitive deficits Degeneration of dopaminergic neurons in nigrostriatal tract causes loss of inhibitory dopamine action on striatal GABAergic neurons and leads to excessive cholinergic neuron excitation of these striatal neurons
Drugs such as levodopa (increases DA activity) can help Whalen K (Ed). Lippincott Illustrated Reviews-Pharmacology 6TH Edn. LLW, 2015. Marc Imhotep Cray MD
97
Vignette 7 A 47-year-old man presents to your clinic complaining of weakness in his hands. He states that he has been frequently dropping objects and is unable to perform fine motor tasks. Physical examination reveals a positive Babinski sign, hyperreflexia, atrophy, diminished strength in the muscles of the hands and calves, and fasciculations. You fear that this patient has a progressive condition, which will ultimately result in his death from respiratory failure in the near future. What is the Diagnosis?
Marc Imhotep Cray MD
98
Amyotrophic Lateral Sclerosis Etiology and Epidemiology: Etiology of sporadic ALS unknown; 5%–10% of cases are familial with autosomal dominant inheritance of a defect on chr 21 or w a defect in SOD-1 (gene product involved in scavenging free radicals); Most commonly affects men over age of 40 Pathology: Progressive disease assoc. w loss of both upper and lower motor neurons Spinal cord: Reduced number of anterior horn neurons w reactive gliosis; degeneration of corticospinal tract neurons Muscle: Neurogenic atrophy w target fibers (fibers w dark center area on cross-section) Marc Imhotep Cray MD
99
ALS (2) Clinical Manifestations: Lower motor neuron signs: Atrophy of muscles; fasciculations Upper motor neuron signs: Hyperreflexia; positive Babinski sign; spasticity Lower and upper motor neuron degeneration tends to present initially with weakness of hands or cramping and spasticity of arms and legs Involvement of respiratory muscles leads to lung infections and eventually death
Treatment and Prognosis: Supportive care Death from respiratory failure usually occurs within 5 years of diagnosis Note: Werdnig-Hoffmann syndrome is an autosomal recessive disease that affects lower motor neurons and is assoc. w degeneration of anterior horns It presents at infancy w tongue fasciculations and “floppy baby” Marc Imhotep Cray MD
100
Amyotrophic lateral sclerosis, gross
Klatt EC. Robbins and Cotran Atlas of Pathology, 3rd Ed. 2015.
Marc Imhotep Cray MD
101
Amyotrophic lateral sclerosis, micro
Klatt EC. Robbins and Cotran Atlas of Pathology, 3rd Ed. 2015.
Marc Imhotep Cray MD
102
Demyelinating Disorders
Marc Imhotep Cray MD
103
Vignette 8 A 27-year-old Caucasian woman presents to your office complaining of visual disturbances. During physical examination, you note that on lateral gaze, one eye does not adduct and the other eye has nystagmus on abduction. Testing of cerebellar function reveals an intention tremor and you also note decreased sensation on both legs. You obtain CSF fluid via a lumbar puncture and find multiple oligoclonal bands of IgG on electrophoresis. You order an MRI of the brain and refer the patient to a neurologist for further care of her condition. What is the Diagnosis?
Marc Imhotep Cray MD
104
Multiple Sclerosis Etiology and Epidemiology: Etiology unknown, although autoimmune, genetic, and environmental factors have been implicated; Incidence increases proportionally with distance from equator and incidence is more common in HLA-DR2 individuals; Most often presents in Caucasian women betw. ages of 20 and 30
Pathology: CNS: Multiple firm plaques representing demyelination within white matter of CNS, especially in optic nerve, brainstem, and periventricular areas Microscopic plaque: Depletion of oligodendrocytes; monocytes, lymphocytes, and lipid-laden macrophages around vessels; gliosis and astrocyte proliferation
Marc Imhotep Cray MD
105
Multiple Sclerosis (2) Clinical Manifestations: Relapsing and remitting course, but eventually remissions become incomplete; classic Charcot triad: nystagmus, scanning speech, and intention tremor; motor and sensory impairment of trunk and extremities (hemiparesis, ataxia); visual impairment (optic neuritis, retrobulbar neuritis, internuclear ophthalmoplegia [on lateral gaze, one eye does not adduct and abducting eye has nystagmus caused by demyelination of MLF]); urinary/bowel incontinence owing to loss of sphincter control Lab findings: Lumbar puncture shows mild lymphocytosis and elevated IgG, manifested as multiple oligoclonal bands on electrophoresis
Treatment: Corticosteroids and other immunosuppressants Marc Imhotep Cray MD
106
Multiple sclerosis, gross
Klatt EC. Robbins and Cotran Atlas of Pathology, 3rd Ed. 2015.
Marc Imhotep Cray MD
107
Multiple sclerosis, MRI
Klatt EC. Robbins and Cotran Atlas of Pathology, 3rd Ed. 2015. Marc Imhotep Cray MD
108
Multiple sclerosis, microscopic
Klatt EC. Robbins and Cotran Atlas of Pathology, 3rd Ed. 2015.
Marc Imhotep Cray MD
109
Vignette 9 A 29-year-old man presents to the emergency room complaining of muscle weakness. He tells you that the weakness began in his calves and has now ascended to involve his thighs, hips, torso, and arms. Upon directed history, you learn that he recently recovered from a flu-like illness. Physical examination reveals symmetrical muscle weakness in all limbs and absent deep tendon reflexes. A lumbar puncture demonstrates an albumino-cytologic dissociation of the CSF. You admit this patient to the intensive care unit for observation as you fear that he may need mechanical respiratory support for his condition. What is the Diagnosis?
Marc Imhotep Cray MD
110
Guillain-Barre Syndrome Etiology: Usually occurs after a flu-like viral illness (eg, EBV, HSV, CMV), but has also been associated with surgical procedures and bacterial infections (mycoplasma, campylobacter) Pathology and Pathophysiology: Pathophysiology: Viral illness causes a T-cell-mediated immune reaction that results in demyelination of peripheral nerves Peripheral nerves: Endoneurial and perivenular infiltration by lymphocytes and macrophages; segmental demyelination
Marc Imhotep Cray MD
111
Guillain-Barre Syndrome (2) Clinical Manifestations: Ascending muscle weakness and paralysis beginning in distal lower limbs; absent deep tendon reflexes; sometimes sensory loss in extremities; facial diplegia; abnormal autonomic function (dysrhythmias, labile blood pressure) Can progress to respiratory failure or become chronic (chronic inflammatory demyelinating polyradiculoneuropathy) Lab findings: Lumbar puncture shows albumino cytologic dissociation of CSF (large protein content increase accompanied by only a mild cell count increase)
Marc Imhotep Cray MD
112
Guillain-Barre Syndrome (3) Treatment and Prognosis: Plasmapheresis; IV immunoglobulin; Supportive care (respiratory support until recovery) Px Most pts recover after weeks to months, but 10%–20% are left w permanent disability DDx: Postinfectious encephalitis can follow viral illnesses (eg, chicken pox, rubella, measles, mumps) and is characterized by transient, widespread demyelination.
Marc Imhotep Cray MD
113
Guillain-BarrĂŠ neuropathy, microscopic
Klatt EC. Robbins and Cotran Atlas of Pathology, 3rd Ed. 2015.
Marc Imhotep Cray MD
114
Demyelination, electron microscopy
Klatt EC. Robbins and Cotran Atlas of Pathology, 3rd Ed. 2015.
Marc Imhotep Cray MD
115
Cerebrovascular Disease
Marc Imhotep Cray MD
116
Vignette 10 A 74-year-old woman is brought to the emergency department after developing left-sided paralysis 1 hour ago. Further evaluation reveals left-sided sensory and motor paralysis, left-sided hyperreflexia, left-sided Babinski reflex, and bilateral symmetric loss of vision in half of her visual fields. When you hear that she has a history of atherosclerosis, you become even more certain of your diagnosis. After obtaining a CT scan of the head to confirm that she does not have a head bleed, you immediately begin to administer thrombolytic therapy. What is the Diagnosis?
Marc Imhotep Cray MD
117
Stroke (CVA) Capsule CVA present clinically as sudden neurological defects and may be caused by intracranial hemorrhage (hemorrhagic stroke) (e.g. subarachnoid or intracranial hemorrhage) or cerebral infarction (usually secondary to thrombotic [ischemic stroke] or embolic occlusion of a carotid or intracranial artery) embolic stroke
Strokes may lead to death or permanent severe neurological defects but modern therapies can result in remarkable clinical recovery
Marc Imhotep Cray MD
118
Ischemic Stroke Etiology: Causes include thrombosis, embolism, dissection, vasculitis, or hypotension Pathology: i.
Cerebral infarction: Associated with thrombosis or embolism Ischemic neuronal change (nuclear pyknosis, eosinophilic cytoplasm) within 12 hours; microglia and monocyte infiltration within 2 days; liquefactive necrosis leading to fluid-filled cavity and reactive astrocytes by 1–3 weeks; gliosis (scar formation) after several months may convert to hemorrhagic infarction, in which blood seeps into infarction and is reabsorbed
Marc Imhotep Cray MD
119
Ischemic Stroke (2) Pathology cont’ed: ii. Watershed infarction: Assoc. w hypotension; see wedge-shaped infarction occurring at edge of area supplied by artery
usually occurs in area betw. ACA and MCA distribution
iii. Lacunar infarcts: Assoc. w hypertension and thrombotic obstruction of small vessels
see small cavitations with surrounding gliosis
Marc Imhotep Cray MD
120
Ischemic Stroke (3) Clinical Manifestations: Depends on site of ischemia and extent of collateral circulation ACA: Sensory loss and weakness in contralateral leg MCA: Contralateral paralysis and sensory loss; homonymous hemianopia (bilateral symmetric loss of vision in half of visual field); aphasias PCA: Contralateral sensory disturbance; macular-sparing homonymous hemianopia Lateral striate arteries: Contralateral paralysis
Marc Imhotep Cray MD
121
Ischemic Stroke (4) Treatment:
Thrombolytic therapy within 3 hours of onset; Antiplatelet therapy (aspirin, dipyridamole); Physical therapy; Statins for cholesterol-lowering effects
Note: Transient Ischemic Attack refers to neurologic deficits caused by cerebral ischemia that resolves within 24 hours suggests pt. is at high risk for having a stroke in near future
Marc Imhotep Cray MD
122
Check-Up Question A 66-year-old Caucasian male with a history of atrial fibrillation develops right-sided weakness and numbness, with his arm more severely affected than his leg. He understands everything that is said to him, but cannot speak. An embolus has most likely occluded which of the following arteries? A. Right middle cerebral artery B. Left anterior cerebral artery C. Left middle cerebral artery D. Anterior inferior cerebellar artery E. Posterior cerebellar artery Source: USMLEWorld, 2015. Neurology/Pathology Q#23.
Marc Imhotep Cray MD
123
Answer C: Left middle cerebral artery This patient displays the symptoms characteristic for a hemispheric stroke. Hemiparesis, with more involvement of the arm than the leg, occurs due to the occlusion of the middle cerebral artery (MCA). This artery supplies the face and arm areas of the motor and sensory cortex, Broca's and Wernicke's speech areas, and the frontal eye field. The MCA also gives rise to the small, penetrating branches that perfuse the internal capsule and basal ganglia. Along with hemiparesis, this patient has Broca's (expressive) aphasia, which manifests with an inability to speak or write, but preserved comprehension of the spoken and written word. Broca's area is located in the inferior frontal gyrus in the dominant (left) hemisphere and is supplied by the left middle cerebral artery. Contrary to popular belief, the left hemisphere of the brain is most often dominant in both right-handed and left-handed people. Educational Objective: Hemiparesis with the arm affected more than the leg occurs due to occlusion of the middle cerebral artery (MCA). If the occluded MCA is in the dominant hemisphere (usually the left), aphasia may also occur. Marc Imhotep Cray MD
124
Acute cerebral ischemia, CT image
Klatt EC. Robbins and Cotran Atlas of Pathology, 3rd Ed. 2015.
Marc Imhotep Cray MD
125
Watershed infarction, gross
Klatt EC. Robbins and Cotran Atlas of Pathology, 3rd Ed. 2015.
Marc Imhotep Cray MD
126
Lacunar infarction, gross
Klatt EC. Robbins and Cotran Atlas of Pathology, 3rd Ed. 2015.
Marc Imhotep Cray MD
127
Cerebral acute infarction, MRI
Klatt EC. Robbins and Cotran Atlas of Pathology, 3rd Ed. 2015.
Marc Imhotep Cray MD
128
Cerebral subacute infarction, gross
Klatt EC. Robbins and Cotran Atlas of Pathology, 3rd Ed. 2015.
Marc Imhotep Cray MD
129
Subacute infarction, microscopic
Klatt EC. Robbins and Cotran Atlas of Pathology, 3rd Ed. 2015.
Marc Imhotep Cray MD
130
Cerebral remote infarction, gross
Klatt EC. Robbins and Cotran Atlas of Pathology, 3rd Ed. 2015.
Marc Imhotep Cray MD
131
Cerebral remote infarction, CT image
Klatt EC. Robbins and Cotran Atlas of Pathology, 3rd Ed. 2015.
Marc Imhotep Cray MD
132
Vignette 11 A 67-year-old man presents to the emergency department after passing out on the sidewalk. When he is revived, he complains of a severe headache and nausea. Past medical history is significant for long-standing hypertension. Physical examination reveals right-sided hemiparesis. You think that this man’s condition may be related to his high blood pressure, which may have caused the formation of CharcotBouchard microaneurysms. A CT scan of the head confirms your suspicions. What is the Diagnosis?
Marc Imhotep Cray MD
133
Hypertension Hypertension is common, often asymptomatic and has many causes including Stress Obesity Renal artery stenosis and Hormonal defects such as Cushing’s syndrome and Conn’s syndrome Chronic hypertension is characterized by an imbalance in sodium and water homeostasis Untreated hypertension can lead to accelerated atherosclerosis and to end-organ damage, including hypertensive nephropathy, hypertensive heart disease and intracerebral hemorrhage Marc Imhotep Cray MD
134
Hemorrhagic Stroke Etiology: Most commonly caused by hypertension; other causes include bleeding disorders, arteriovenous malformations, brain tumors, or amyloid angiopathy Pathology and Pathophysiology: Pathophysiology Chronic hypertension is associated with CharcotBouchard microaneurysms are usually located within basal ganglia Rupture of these aneurysms may be proximal cause of hemorrhage Brain: Hemorrhage usually located in basal ganglia or thalamus; central area of blood surrounded by edematous brain tissue → edema resolves and reactive astrocytes and macrophages appear at edge of the injury → gliosis
Marc Imhotep Cray MD
135
Hemorrhagic Stroke (2) Clinical Manifestations: Impairment of consciousness; nausea and vomiting; headache; neurologic deficits (especially hemiparesis and sometimes hemisensory disturbance)
Treatment: Reverse any coagulopathies Strict blood pressure control surgical decompression if necessary for large intracranial hemorrhage
Marc Imhotep Cray MD
136
Cerebral hypertensive hemorrhage, gross
Klatt EC. Robbins and Cotran Atlas of Pathology, 3rd Ed. 2015.
Marc Imhotep Cray MD
137
Vascular (multi-infarct) dementia, gross Multiple vascular events, including embolic arterial occlusion, atherosclerosis with vascular narrowing and thrombosis, and hypertensive arteriolar sclerosis may lead to focal but additive loss of cerebral tissue Cumulative effect of multiple small areas of infarction ( ) may result in clinical findings equivalent to AD along with focal neurologic deficits or gait disturbances Vascular dementia marked by loss of higher mental function in a stepwise, not continuous, fashion Marc Imhotep Cray MD
Klatt EC. Robbins and Cotran Atlas of Pathology, 3rd Ed. Philadelphia: Saunders, 2015. 138
Cerebral hypertensive hemorrhage, CT image
Klatt EC. Robbins and Cotran Atlas of Pathology, 3rd Ed. 2015.
Marc Imhotep Cray MD
139
Vignette 12 A 44-year-old woman presents to the emergency department complaining of nausea and the worst headache of her life. Upon further questioning, you learn that she is a heavy smoker and she has a history of poorly controlled hypertension. You decide to perform a lumbar puncture, which reveals blood in the CSF. A CT scan of the head demonstrates blood in the basal cisterns. You immediately admit her to the hospital for a cerebral angiography in order to evaluate for the best treatment of her condition. What is the Diagnosis?
Marc Imhotep Cray MD
140
Berry Aneurysm and Subarachnoid Hemorrhage Etiology and Epidemiology: Causes of subarachnoid hemorrhage include rupture of berry aneurysm, trauma, and arteriovenous malformation Most berry aneurysms occur sporadically, but risk factors include hypertension, cigarette smoking, coarctation of aorta, APKD, connective tissue disorders, and neurofibromatosis type 1 Rupture of berry aneurysms occurs more frequently in women and in those over the age of 40
Pathology: Subarachnoid hemorrhage: Blood in subarachnoid space; fibrosis, occurring after resolution, may lead to CSF obstruction Berry aneurysm: Often occur at arterial bifurcations of circle of Willis; outpouching of arterial wall with intimal thickening and media thinning at neck of aneurysm; media is absent in sac wall Marc Imhotep Cray MD
141
Berry Aneurysm and Subarachnoid Hemorrhage (2) Clinical Manifestations: Subarachnoid hemorrhage: “Worst headache of my life;” N & V; loss of consciousness, may have fever or nuchal rigidity, can be fatal Lab findings: Lumbar puncture reveals blood in CSF Imaging: CT scan demonstrates blood in basal cisterns
Treatment: Surgical repair; supportive care; therapeutic approach depends on cerebral angiography Note: Arteriovenous malformations are congenital vascular malformations usually localized to subarachnoid space, but may extend into brain tissue manifest clinically in young adults as seizures or hemorrhage Marc Imhotep Cray MD
142
Berry aneurysm, gross
Klatt EC. Robbins and Cotran Atlas of Pathology, 3rd Ed. 2015.
Marc Imhotep Cray MD
143
Berry aneurysm, angiogram
Klatt EC. Robbins and Cotran Atlas of Pathology, 3rd Ed. 2015.
Marc Imhotep Cray MD
144
Subarachnoid hemorrhage, gross
Klatt EC. Robbins and Cotran Atlas of Pathology, 3rd Ed. 2015.
Marc Imhotep Cray MD
145
Vascular malformation, gross
Klatt EC. Robbins and Cotran Atlas of Pathology, 3rd Ed. 2015.
Marc Imhotep Cray MD
146
Vignette 13 A 78-year-old woman is brought to the emergency department by her son because of headaches and altered mental status. He tells you that the patient fell down the stairs 2 weeks ago, but that she appeared fine immediately after the fall. Physical examination reveals bilateral papilledema. When a CT scan of the head reveals a 3-cm crescent-shaped collection of fluid on the right side of the head that crosses suture lines with a 7-mm midline shift, you suspect that her current condition is related to tearing of the bridging veins between the cerebrum and venous sinuses in the dura and you schedule her for immediate surgical drainage of the blood. What is the Diagnosis? Marc Imhotep Cray MD
147
Epidural and Subdural Hematoma Etiology: Epidural hematoma: Caused by tearing of middle meningeal artery, (middle meningeal vein, or dural sinus) which is often caused by skull fracture Subdural hematoma: Caused by tearing of bridging veins located betw. cerebrum and venous sinuses in dura mater often owing to head injury Pathology: Epidural: Accumulation of blood betw. dura and skull leading to cerebral compression Subdural: Accumulation of blood betw. dura and arachnoid bleeding is self-limited, but hematoma can grow owing to osmotic movement of water resolution with granulation tissue can occur as well leading to a chronic subdural hematoma Marc Imhotep Cray MD
148
Epidural and Subdural Hematoma (2) Clinical Manifestations: Epidural: Loss of consciousness (LOC) , followed by lucid period, followed by headache, altered mental status, seizures, focal neurologic deficits, and eventually coma o Imaging: Head CT shows lens shaped /convex that does not cross suture lines Subdural: Headache; altered mental status; other signs of cerebral compression; clinical signs occur gradually, appearing hours to weeks after injury o Imaging: Head CT shows crescent-shaped/ concave disk that crosses suture lines Treatment: Surgical drainage of blood; reversal of coagulopathy Marc Imhotep Cray MD
149
Epidural hematoma, gross
Klatt EC. Robbins and Cotran Atlas of Pathology, 3rd Ed. 2015.
Marc Imhotep Cray MD
150
Epidural hematoma, CT image
Klatt EC. Robbins and Cotran Atlas of Pathology, 3rd Ed. 2015.
Marc Imhotep Cray MD
151
Subdural hematoma, gross, and bridging veins, gross
Klatt EC. Robbins and Cotran Atlas of Pathology, 3rd Ed. 2015. Marc Imhotep Cray MD
152
Subdural hematomas, CT images
Klatt EC. Robbins and Cotran Atlas of Pathology, 3rd Ed. 2015.
Marc Imhotep Cray MD
153
Comparison of epidural & subdural hemorrhages
Le, T; Krause, K (Eds.) First AID for Basic Sciences: Organ Systems. Mc Graw-Hill, 2009, Pg.469, Fig. 6-10.
Marc Imhotep Cray MD
154
Intracranial hemorrhage Capsular Summary Epidural hematoma Lucid interval in 50% (“talk and die” syndrome) Subdural hematoma More common than epidural Always cause brain damage Elderly, diabetic, atrophy May be associated with contusion, subarachnoid, and hemorrhage Subarachnoid hemorrhage Ruptured berry aneurysm (Assoc. w APCKD) “Worst headache of my life” Marfan, Ehlers-Danlos type 4 Apolycystic disease, HTN, smoking, blacks, incr. age Intracerebral/parenchymal hemorrhage HTN, trauma, infarct Amyloid angiopathy, DM Charcot Bouchard aneurysm from HTN Basal ganglia thalamus Marc Imhotep Cray MD
155
Intracranial hemorrhage Check-Up
Make the Diagnosis: See First AID for the USMLE Step 1, 2020, Pgs.513 for details. Marc Imhotep Cray MD
156
UWorld & Epidural vs Subdural Hematoma vs SAH
USMLEWorld, 2015. Neurology /Pathology Q#3. Click graphics for respective question plates.
Marc Imhotep Cray MD
157
Other Disorders
Marc Imhotep Cray MD
158
Vignette 14 A 43-year-old woman presents to the emergency room complaining of episodic loss of vision. She also reports having had severe headaches associated with nausea and vomiting over the past month. Physical examination reveals bilateral papilledema and a CT scan of the head demonstrates dilation of the ventricular system of the brain. You suspect that she may need placement of a ventriculoperitoneal shunt to treat her condition and you admit her to the hospital to obtain neurosurgical consultation. What is the Diagnosis?
Marc Imhotep Cray MD
159
Hydrocephalus Etiology: Caused by accumulation of increased volume of CSF within cranium can either result from obstruction to CSF circulation (attributed to tumors or inflammation) or from overproduction of CSF by tumors of choroid plexus Pathology Gross: Dilation of ventricles Four variants of hydrocephalus: 1. 2. 3. 4.
Internal: excessive CSF is present only in ventricular system External: excessive CSF is present only in subarachnoid space Communicating: CSF flows freely betw. ventricles and subarachnoid space; Noncommunicating: CSF flow betw. ventricles and subarachnoid space is obstructed
Marc Imhotep Cray MD
160
Hydrocephalus (2) ď ąClinical Manifestations: May present w enlargement of skull in adults, seizures, headaches, visual disturbances, nausea and vomiting (N/V), and other signs of increased intracranial pressure (ICP) ď ąTreatment: Insertion of ventriculoperitoneal shunt; removal of obstruction or choroid plexus tumor
Note: Hydrocephalus ex vacuo refers to dilation of ventricles w an increase in CSF volume resulting from a loss of brain tissue (often by infarction or Alzheimer disease)
Marc Imhotep Cray MD
161
Hydrocephalus, gross
Klatt EC. Robbins and Cotran Atlas of Pathology, 3rd Ed. 2015.
Marc Imhotep Cray MD
162
Hydrocephalus, CT image
Klatt EC. Robbins and Cotran Atlas of Pathology, 3rd Ed. 2015.
Marc Imhotep Cray MD
163
Hydrocephalus ex vacuo, gross
Klatt EC. Robbins and Cotran Atlas of Pathology, 3rd Ed. 2015.
Marc Imhotep Cray MD
164
Vignette 15 A 25-year-old woman presents to your office complaining of diminished sensation in both arms. Further neurologic evaluation of her arms reveals that she has diminished pain and temperature sensation, but that her touch sensation and proprioception are intact. She demonstrates no other neurologic deficits. When an MRI reveals cystic dilation in the center of the cervical spinal cord, you refer her to a neurosurgeon for treatment of her condition. What is the Diagnosis?
Marc Imhotep Cray MD
165
Syringomyelia Etiology: Often assoc. w Arnold-Chiari malformation; also caused by intraspinal neoplasms or trauma Pathology:
Spinal cord: Formation of fluid-filled cavity often extending from central canal usually in cervical region of cord (most commonly C7T1)results in destruction of adjacent gray and white matter (crossing fibers of spinothalamic tract) w resultant reactive gliosis
Clinical Manifestations: Loss of pain and temp. of upper extremities w preservation of touch and proprioception; may progress to involve motor and other sensory tracts Treatment: Surgical drainage of cavity Marc Imhotep Cray MD
166
Vignette 16 A 32-year-old woman presents to your office complaining of intermittent double vision. She tells you that this tends to occur in the afternoons and evenings. Physical examination reveals ptosis of both eyes that worsens when the patient is asked to actively keep her eyelids elevated. When her symptoms markedly improve after edrophonium administration, you wonder if she may also have a thymoma. What is the Diagnosis?
Marc Imhotep Cray MD
167
Myasthenia Gravis Etiology and Epidemiology: Caused by antibodies directed against acetylcholine (ACh) receptors at neuromuscular junction Presents most frequently in women under age of 40 Pathology and Pathophysiology: Neuromuscular junction: Loss of ACh receptors; infiltration of immune complexes and complement factors Pathophysiology: Antibodies lead to Ach receptor degradation causing a virtual block of synaptic transmission
Marc Imhotep Cray MD
168
Myasthenia Gravis (2) Clinical Manifestations: Muscle weakness that worsens with fatigue; common initial presentation is ptosis or diplopia owing to extraocular (EOM) muscle involvement, but also involves muscles of extremities and facial muscles Diagnosis confirmed by improvement after administration of a short-acting anticholinesterase (edrophonium) Assoc. w thymoma or thymic hyperplasia Lab findings: Antibodies to ACh receptors Treatment: Anticholinesterase drugs (ie, pyridostigmine); thymectomy; immunosuppression; plasmapheresis if severe flare Marc Imhotep Cray MD
169
Normal thymus, microscopic
Klatt EC. Robbins and Cotran Atlas of Pathology, 3rd Ed. 2015.
Marc Imhotep Cray MD
170
Thymic hyperplasia, microscopic
Klatt EC. Robbins and Cotran Atlas of Pathology, 3rd Ed. 2015.
Marc Imhotep Cray MD
171
Vignette 17 A 21-year-old man presents to the emergency department complaining of a severe headache. Physical examination reveals a fever to 1020F, nuchal rigidity, and photophobia. You perform a lumbar puncture, which initially reveals purulent CSF infiltrated with neutrophils, increased protein content, and decreased glucose content. While you await culture results, you admit the patient to the hospital and begin empiric broad spectrum antibiotics to treat his condition. What is the Diagnosis?
Marc Imhotep Cray MD
172
Pyogenic and Viral Meningitis Pyogenic Etiology Pyogenic meningitis: Causes include group B streptococci, E coli, Listeria in neonates and infants; H influenzae and N meningitidis in children and young adults; pneumococcus, Listeria, and gram-negative rods in older adults Pathology Pyogenic meningitis: Purulent exudate within leptomeninges; engorged meningeal vessels; neutrophils within subarachnoid space Viral Viral meningitis: Causes include HSV virus, Coxsackie virus, echoviruses, and arboviruses Viral meningitis: There may be no abnormality or a mild lymphocytic infiltrate in subarachnoid space; mild edema may be present Marc Imhotep Cray MD
173
Pyogenic and Viral Meningitis (2) Clinical Manifestations Pyogenic meningitis: Headache; photophobia; neck stiffness; fever; irritability. Lab findings: Lumbar puncture shows cloudy CSF with neutrophils, increased protein, decreased glucose and increased opening pressure
Viral meningitis: Headache; photophobia; neck stiffness; fever; irritability. Lab findings: Lumbar puncture shows lymphocytosis, mildly elevated protein, and normal glucose
Treatment: Pyogenic meningitis: Antibiotics and supportive care Viral meningitis: Self-limiting; acyclovir for HSV meningitis Marc Imhotep Cray MD
174
Aseptic meningitis Aseptic meningitis can be caused by drugs and can also be a manifestation of certain rheumatologic or other systemic disorders A 45-year-old day care provider presents in August with a fever of 38°C, headache, photophobia, and neck stiffness. A clinical diagnosis of meningitis is made. Based on the epidemiology of meningitis in the United States, what is the most likely cause of her disease? A. Cryptococcus neoformans B. Haemophilus influenzae C. Neisseria meningitidis D. Streptococcus pneumoniae E. Viral Runge MS and Greganti MA. Netter's Internal Medicine 2nd Ed. Saunders 2008. Marc Imhotep Cray MD
175
Answer is E: Viral Overall viruses are most common cause of meningitis Viral infection of meninges is known as aseptic meningitis less severe than bacterial meningitis Whereas viral meningitis may not require hospitalization, bacterial meningitis is a medical emergency requiring prompt Dx and admin. of empiric ABXs To avoid unnecessary hospitalization and admin. of ABXs essential to be able to distinguish betw. septic (bacterial) and aseptic (viral) meningitis o CSF findings help make this distinction Enteroviruses--specifically coxsackieviruses and echoviruses-- cause greatest number of cases of aseptic meningitis common infections of children and adults w close contact w children show a seasonal occurrence, w most cases in summer and early fall Related: AAP Updates Guidelines for Evaluating Simple Febrile Seizures in Children (American Family Physician, Volume 83, Number 11 , June 1, 2011.); Febrile Seizures in Children (Osmosis)
S. pneumoniae and N. meningitidis are important cause of meningitis in both children and adults o Vaccination has decr. occurrence of these two in U.S. and Western societies C. neoformans is a cause of meningitis in immune suppressed individuals Marc Imhotep Cray MD
176
Meningitis (More discussion‌) A Vignette and Clinicopathologic Capsule
Marc Imhotep Cray MD
177
Vignette 18 An 18-year-old girl is brought to the college emergency room by her roommate. The roommate 4 claimed that the patient had been feeling fine the night before but this morning had a high fever and was difficult to arouse. On physical examination, the patient was found to have a temperature of 102°F, to be very lethargic, and to have a petechial rash. Examination of her cerebrospinal fluid revealed numerous neutrophils and gram-negative diplococci. Her records indicated that she had received the tetravalent meningitis vaccine before graduating from high school.
Marc Imhotep Cray MD
178
Etiopathogenesis Pts w meningitis can present acutely or chronically a distinction that helps determine likely etiologies Most common acute presentations result from bacterial and aseptic meningitis Aseptic meningitis may occur in pts. w viral infections or in assoc. w an adverse drug reaction (ADR)
A subacute picture--CSF pleocytosis persists for longer than 4 weeks-- is more likely to be assoc. w fungal or vasculitic meningitis Marc Imhotep Cray MD
179
Acute meningitis, gross
Klatt EC. Robbins and Cotran Atlas of Pathology, 3rd Ed. 2015.
Marc Imhotep Cray MD
180
Acute meningitis, microscopic
Klatt EC. Robbins and Cotran Atlas of Pathology, 3rd Ed. 2015.
Marc Imhotep Cray MD
181
Acute meningitis, MRI
Klatt EC. Robbins and Cotran Atlas of Pathology, 3rd Ed. 2015.
Marc Imhotep Cray MD
182
Meningitis in a Ethiopian child w a very rigid neck  Cerebral malaria should be in DDx for this child
 Most bacterial meningitis in children can now be prevented by vaccines frequently still not available in developing countries
Marc Imhotep Cray MD
Usatine RP, et al. The Color Atlas of Family Medicine. New York: McGraw-Hill, 2013.
183
Risk Factors Meningitis: Risk Factors (HE IS Chief Of SPAIN)
Head trauma Extreme age Immunocompromised state Sinusitis Cancer Otitis Splenectomy Parameningeal infection Alcoholism Infections (systemic, especially respiratory) Neurosurgical procedures
Marc Imhotep Cray MD
184
Most Common Organisms & Likeliest Bug in Age Group Meningitis: Most Common Organisms (SIN) Streptococcus pneumoniae Influenzae (Haemophilus) Neisseria meningitidis Pyogenic Meningitis: Likeliest Bug in Age Group (In order from death to birth)
Streptococcus pneumoniae (elderly/>65) Neisseria meningitis (young adults) Haemophilus influenzae (older infants, kids) Escherichia coli (infants) Mnemonic= She Never Had Entertainment
Marc Imhotep Cray MD
185
Most Common Organisms Haemophilus influenzae used to be most common cause of meningitis in newborns but is now only rarely seen in this age group. Why has this changed? Newborns are now given a vaccine to protect them against H. influenzae type b however, vaccine is effective only for about first 2 years of life
In a patient w human immunodeficiency virus (HIV), what infective agents may be more likely to cause meningitis than in a patient who has a fully competent immune system? In a pt. w HIV, opportunistic infections such as Toxoplasmosis, Cryptococcus, and Human polyomavirus 2 (JC virus) must be considered in differential diagnosis (DDx) o Remember JC virus-induced Progressive Multifocal Leukoencephalopathy (a demyelinating disease) Marc Imhotep Cray MD
186
Clinical Manifestations Signs and Symptoms: Classic triad of bacteria meningitis consists of following: Fever Headache Neck stiffness Other symptoms can include N/V, photophobia, sleepiness, confusion, irritability, delirium, and coma Pts with viral meningitis may have a history of preceding systemic symptoms (eg, myalgias, fatigue, or anorexia)
Marc Imhotep Cray MD
187
Sn & Sx cont. Ability to mount an immune responses must be kept in mind when assessing severity of symptom Immunocompromised pts. may have a more subtle presentation yet they are at very high risk for poor outcome In bacterial meningitis, temp. usually exceeds 37.7º C (99.9º F) Temp. conversion formulas: °C x 9/5 + 32 = °F (°F - 32) x 5/9 = °C
Low-grade fever is more often present in viral meningitis o Fever may be entirely absent in immunocompromised patients
Marc Imhotep Cray MD
188
Sn & Sx cont. Mental status changes occur in bacterial meningitis in 44% of cases but are found in only 3% of viral meningitis cases Seizures occur in range of 20% to 25% of pts w bacterial meningitis focal findings such as CN deficits are even more common, occurring in 25% to 30% of these pts. Neck stiffness is a specific sign and has a sensitivity of about 70%
Marc Imhotep Cray MD
189
Physical Examination Meningeal signs, most commonly meningismus, are present in about 88% of cases of bacterial meningitis A constellation of Sn & Sx (e.g., headache, neck stiffness) suggestive of meningitis meningeal irritation without objective findings
Other classic signs are Kernig and Brudzinski signs Kernig sign is pain in back upon passive extension of one leg at knee and thigh Brudzinski sign is flexion of legs at thighs when patient’s neck is flexed
Jolt accentuation of headache is a very sensitive finding for meningitis Elicited by having patient turn head rapidly horizontally a number of times per second to assess for worsening of headache Marc Imhotep Cray MD
190
Kernig and Brudzinski Signs
Runge MS and Greganti MA. Netter's Internal Medicine 2nd Ed. Saunders 2008.
Kernig sign is performed by flexing hip w knee flexed and then having pt. extend at knee while keeping hip flexed o If pt. is reluctant to fully extend b/c of nuchal discomfort, test is positive
Brudzinski sign is performed by passively flexing neck while patient is supine o test is considered to be positive if pt. spontaneously flexes hips
Both tests assess for nuchal rigidity secondary to meningeal inflammation Marc Imhotep Cray MD
191
PE cont. Thorough neurologic examination should be performed, w attention given to accurate assessment of:
level of consciousness presence or absence of cranial nerve deficits assessment for papilledema, and documentation of any focal motor or sensory defects
Skin should be carefully examined for lesions Purpura strongly suggests meningococcal disease Petechiae almost as frequently seen as purpura in meningococcal meningitis can also occur in rickettsial diseases and sometimes in pneumococcal meningitis
Marc Imhotep Cray MD
192
Diagnosis
Runge MS and Greganti MA. Netter's Internal Medicine 2nd Ed. Saunders 2008.
Marc Imhotep Cray MD
193
Meningitis Dx based on CSF findings Cerebrospinal Fluid Findings In Meningitis Infective Agent Bacterial Viral Fungal
WBC Differential Normal Normal/
Cell Type
Protein
PMNs Lymphocytes Normal Lymphocytes
Glucose
Normal
Opening Pressure Normal/
Redrawn after: Brown TA, Shah SJ. USMLE Step 1 Secrets 3rd ed. Sanders, 2013; Pg. 672, Table 21-26.
Marc Imhotep Cray MD
194
Meningitis Dx based on CSF findings (2)
Runge MS and Greganti MA. Netter's Internal Medicine 2nd Ed. Saunders 2008.
Marc Imhotep Cray MD
195
Neisseria meningitidis Organism: N meningitidis Phys Char: Gram-negative diplococcus, 13 serogroups based on polysaccharide capsule of which A, B, C, Y, W135 are most important Etio and Epidem: major mode of transmission is by respiratory droplets from carriers Outbreaks can arise when carriers and susceptible individuals are brought together under crowded conditions university dormitories and military barracks
Marc Imhotep Cray MD
196
Neisseria meningitidis (To Osmosis Vid.) Clinical Findings: Nasopharynx is portal of entryďƒ from nasopharynx organisms may reach bloodstream (meningococcemia)  Meningitis is most common complication of meningococcemia o Widespread petechiae and ecchymoses are signs of meningococcemia o Severe cases can lead to disseminated intravascular coagulation (DIC)
Marc Imhotep Cray MD
197
Neisseria meningitidis cont. Pathogenesis: Major virulence factors is an antiphagocytic polysaccharide capsule, endotoxin (lipooligosaccharide), IgA1 protease, and pili As w N gonorrhoeae, complement system is important in immune clearance through classical pathway o Individuals with complement deficiencies are at higher risk for dissemination
Laboratory: Latex agglutination tests used to Dx N meningitidis in CSF, as are direct Gram stains Latex agglutination is more sensitive than Gram stain of CSF however, it is not as sensitive as culture, and a negative test does not rule out an infectious cause of patient’s symptoms o N meningitidis can be grown on blood or chocolate agar
Marc Imhotep Cray MD
198
Neisseria meningitidis cont. Treatment: In general, penicillin or third-generation cephalosporins are effective for treatment Often a combination of IV vancomycin and ceftriaxone is used b/c of their central nervous system (CNS) penetration and broad coverage Appropriate time to initiate antibiotic therapy and what antimicrobial agent could be used ATB Tx must be initiated immediately when bacterial meningitis is suspected Based on age of pt. and morphology on Gram stain an appropriate agent can be chosen AEs: IV vancomycin nephrotoxicity and ototoxicity, thrombophlebitis, Red man syndrome (prevented by antihistamines) Ciprofloxacin or rifampin is used for high-risk individuals who have been in close contact with an index case Marc Imhotep Cray MD
199
Neisseria meningitidis cont. Prevention A vaccine against serogroups A, C, Y, & W135 is available B serogroup polysaccharide is a poor immunogen and is therefore not in current vaccine o A high percentage of cases involve serogroup B
Chemoprophylaxis: A single dose of 500 mg of ciprofloxacin is preferred regimen for prophylaxis of adults against Neisseria meningitidis Rifampin can also be used Children can be protected using a single dose of ceftriaxone as fluoroquinolones contraindicated in children Damages cartilage in young children
Marc Imhotep Cray MD
200
Question What adrenal disease should be suspected in a young patient with bacterial meningitis due to Neisseria meningitidis who also becomes acutely hypotensive? Sn & Sx: septicemia, hypotension disseminated intravascular coagulation (DIC) adrenal hemorrhage, and petechial rash Marc Imhotep Cray MD
Raff RB, Rawls SM, Beyzarov EP. Netter's Illustrated Pharmacology, Updated Edn. Saunders, 2014. 201
Waterhouse-Friderichsen syndrome, gross Ans. Waterhouse-Friderichsen syndrome typically causes bilateral adrenal hemorrhage, which can be rapidly fatalďƒ acute adrenal cortical insufficiency syndrome Responsible bacterium is Neisseria meningitidis
Klatt EC. Robbins and Cotran Atlas of Pathology, 3rd Ed. Saunders, 2015.
Marc Imhotep Cray MD
202
Vignette 19 A 10-year-old boy is brought to your pediatric neurology clinic by his parents who have noticed that he has selflimited periods of unresponsiveness, even though his eyes are open. When questioned, the patient states that he sees flashes or blinking lights at times and has momentary lapses of “forgetfulness.� The parents report that their other child had similar episodes, but the episodes subsided after puberty. You inform the parents that you believe their child may have a mild form of epilepsy. You order an EEG, which you expect will demonstrate a classic 3-Hz spike-and-wave EEG pattern. What is the Diagnosis?
Marc Imhotep Cray MD
203
Seizures Disorders Capsule Etiology: Causes for seizures include toxins (ie, drugs, alcohol withdrawal), intracranial pathology (stroke, bleed, tumor, infection, degenerative disorders), metabolic abnormalities (hyponatremia, hypoglycemia) or epilepsy (syndrome of recurrent seizures) Pathophysiology: Abnormal discharge of CNS neurons results in neurological symptoms
Marc Imhotep Cray MD
204
Seizures Capsule cont. ď ąClinical Manifestations: Usually preceded by aura (odd smell/vision), then followed by seizure; seizure often followed by postictal period (mins. to hrs. of resolving confusion and lethargy)  Seizures may be either partial (involving discrete area of brain) or generalized and include (1) Simple partial: no impairment of consciousness; involves motor, sensory, or autonomic brain; (2) Complex partial: similar to simple partial except that consciousness impaired; (3) Tonic-clonic (grand mal): contraction of muscles alternating with relaxation; (4) Absence: lapse of consciousness without loss of postural tone; (5) Myoclonic: sudden, brief contractions
Marc Imhotep Cray MD
205
Overview of Epilepsy Approximately 10% of population will have at least one seizure in their lifetime Epilepsy vs Seizures A seizure is a single occurrence, whereas epilepsy is a neurological condition characterized by two or more unprovoked seizures
Globally, epilepsy is third most common neurologic disorder after Cerebrovascular Disease and Alzheimer’s disease Epilepsy is not a single entity but an assortment of different seizure types and syndromes originating from several mechanisms that have in common sudden, excessive, and synchronous discharge of cerebral neurons Marc Imhotep Cray MD
206
Overview of Epilepsy (2) Abnormal electrical activity may result in a variety of events, including loss of consciousness (LOC), abnormal movements, atypical or odd behavior, and distorted perceptions that are of limited duration but recur if untreated Site of origin of abnormal neuronal firing determines symptom produced For example, if motor cortex is involved, patient may experience abnormal movements or a generalized convulsion Seizures originating in parietal or occipital lobe may include visual, auditory, and olfactory hallucinations
Marc Imhotep Cray MD
207
Electroencephalography EEG permits recording of collective electrical activity of cerebral cortex as a summation of activity measured as a difference between two recording electrodes Recording electrodes (leads) are placed on the scalp on at least 16 standard sites, and recordings of potential differences between key Raff RB, Rawls SM, Beyzarov EP. Netter's Illustrated Pharmacology, electrodes are obtained Updated Edition. Philadelphia: Sanders, 2014. “The diagnosis of epilepsy is often not straightforward, and misdiagnosis is not rare. A detailed and reliable account of the event by an eyewitness is the most important part of the diagnostic evaluation, but may not be available. Electroencephalography (EEG) is an important diagnostic test in evaluating a patient with possible epilepsy. It can provide support for the diagnosis of epilepsy and also assists in classifying the underlying epileptic syndrome. However, there are several reasons why EEG alone cannot be used to make or refute a specific diagnosis of epilepsy: ●Most EEG patterns can be caused by a wide variety of different neurologic diseases. ●Many diseases can cause more than one type of EEG pattern.” (From https://www.uptodate.com/contents/electroencephalography-eeg-in-the-diagnosis-of208 Marc Imhotep Cray MD seizures-and-epilepsy)
EEG (2) Examples are provided of a 1. normal EEG taken when client is awake with eyes closed and 2. normal EEG sleeping Abnormal patterns of activity can be seen in presence of 3. tumors and 4. seizures
Raff RB, Rawls SM, Beyzarov EP. Netter's Illustrated Pharmacology, Updated Edition. Philadelphia: Sanders, 2014.
Marc Imhotep Cray MD
209
Etiology of Epilepsy and Seizures In most cases, epilepsy has no identifiable cause Focal areas that are functionally abnormal may be triggered into activity by changes in physiologic factors, such as alteration in blood gases, pH, electrolytes, and blood glucose and changes in environmental factors, such as sleep deprivation, alcohol intake, flashing lights and stress Neuronal discharge results from firing of a small population of neurons in a specific area of brain referred to as the “primary focus”
Neuroimaging techniques, such as magnetic resonance imaging (MRI), positron emission tomography (PET) scans, and single photon emission coherence tomography (SPECT) may identify areas of concern Marc Imhotep Cray MD
210
Etiology of Seizures (2) Intracranial:
Raff RB, Rawls SM, Beyzarov EP. Netter's Illustrated Pharmacology, Updated Edition. Philadelphia: Sanders, 2014.
Marc Imhotep Cray MD
211
Etiology of Seizures (3) Extracranial:
Raff RB, Rawls SM, Beyzarov EP. Netter's Illustrated Pharmacology, Updated Edition. Philadelphia: Sanders, 2014.
Marc Imhotep Cray MD
212
Classification of Seizures It is important to correctly classify seizures to determine appropriate treatment Seizures have been categorized by site of origin, etiology, electrophysiologic correlation, and clinical presentation The nomenclature developed by International League Against Epilepsy is considered standard way to classify seizures and epilepsy syndromes Seizures have been classified into two broad groups: focal and generalized Marc Imhotep Cray MD
213
Framework for classification of epilepsies_ILAE (International League Against Epilepsy)
Source: Scheffer, et.al. Epilepsia, 58(4):512–521, 2017 (It’s in the repository.) Marc Imhotep Cray MD
214
A. Focal Seizures, Simple complex Involve only a portion of brain, part of one lobe of one hemisphere impaired consciousness, automatisms
Symptoms depend on site of neuronal discharge and on extent to which electrical activity spreads to other neurons in brain Focal seizures may progress to become generalized tonic-clonic seizures
Marc Imhotep Cray MD
215
Focal Seizures, Simple partial Caused by a group of hyperactive neurons exhibiting abnormal electrical activity and confined to a single locus in brain Electrical discharge does not spread, and patient does not lose consciousness or awareness Pt. often exhibits abnormal activity of a single limb or muscle group controlled by region of brain experiencing disturbance Pt. may also show sensory distortions May occur at any age Marc Imhotep Cray MD
216
Raff RB, Rawls SM, Beyzarov EP. Netter's Illustrated Pharmacology, Updated Edition. Philadelphia: Sanders, 2014. Marc Imhotep Cray MD
217
Focal Seizures, Complex partial Exhibit complex sensory hallucinations and mental distortion Motor dysfunction may involve chewing movements, diarrhea, and/or urination Consciousness is altered Simple partial seizure activity may spread to become complex and then spread to a secondarily generalized convulsion May occur at any age Marc Imhotep Cray MD
218
Raff RB, Rawls SM, Beyzarov EP. Netter's Illustrated Pharmacology, Updated Edition. Philadelphia: Sanders, 2014. Marc Imhotep Cray MD
219
B. Generalized Seizures May begin locally and then progress to include abnormal electrical discharges throughout both hemispheres of brain Primary generalized seizures may be convulsive or nonconvulsive, and
Patient usually has an immediate LOC
Marc Imhotep Cray MD
220
Generalized Seizures, 1. Tonic-Clonic (Grand Mal seizures) These seizures result in LOC, followed by tonic (continuous contraction) and clonic (rapid contraction and relaxation) phases “Ictal phase”
May be followed by a period of confusion and exhaustion due to depletion of glucose and energy stores “Postictal phase”
Marc Imhotep Cray MD
221
Grand Mal seizures
Raff RB, Rawls SM, Beyzarov EP. Netter's Illustrated Pharmacology, Updated Edition. Philadelphia: Sanders, 2014.
Marc Imhotep Cray MD
222
Status Epilepticus An unbroken cycle of seizures
Raff RB, Rawls SM, Beyzarov EP. Netter's Illustrated Pharmacology, Updated Edition. Philadelphia: Sanders, 2014.
Status epilepticus is continuous tonic-clonic seizure for over 30 minutes Complications include anoxic brain injury and lactic acidosis Marc Imhotep Cray MD
223
Generalized Seizures, 2. Absence (Petit Mal seizures) These seizures involve a brief, abrupt, and self-limiting loss of consciousness Onset generally occurs in patients at 3 to 5 years of age and lasts until puberty or beyond Patient stares and exhibits rapid eye-blinking, which lasts 3 to 5 seconds
Has a very distinct three-per-second spike and wave discharge seen on electroencephalogram
Marc Imhotep Cray MD
224
Petit Mal seizures (Absence Seizures)
3 Hz spike-and-wave discharges, no postictal confusion, blank stare Raff RB, Rawls SM, Beyzarov EP. Netter's Illustrated Pharmacology, Updated Edition. Philadelphia: Sanders, 2014.
Marc Imhotep Cray MD
225
Generalized Seizures, 3. Myoclonic Consist of short episodes of muscle contractions that may recur for several minutes Exhibit as brief jerks of limbs (quick, repetitive jerks) Occur at any age but usually begin around puberty or early adulthood
Marc Imhotep Cray MD
226
Generalized Seizures cont. 4. Clonic: Consist of short episodes of muscle contractions that may closely resemble myoclonic seizures  Consciousness is more impaired compared to myoclonic
5. Tonic: Involve increased tone in extension muscles and are generally less than 60 seconds long 6. Atonic: Also known as drop attacks and are characterized by a sudden loss of muscle tone
Marc Imhotep Cray MD
227
Epilepsy (generalized, focal) - tonic-clonic, tonic, clonic, causes, symptoms-Osmosis
Le T ; Bhushan V. First Aid for the USMLE Step 1 2020. McGraw-Hill, 2020, 517.
Marc Imhotep Cray MD
228
Neoplasms What are the common presenting features of patients with brain tumors? Patients may present with a variety of symptoms including nausea, headache, seizures, focal findings (eg, compression of a single cranial nerve), and/or altered mental status/confusion. What is the most common adult brain tumor? Metastases (eg, lung, breast, melanoma) Within the skull, where are adult brain tumors most often located? Superior to the tentorium—“supratentorial” Within the skull, where are pediatric brain tumors most often located? Inferior to the tentorium—”infratentorial” What is the most common primary brain tumor in adults? Glioblastoma multiforme (GBM) What is a “glioma”? A glioma is a relatively nonspecific term applied to any brain tumor derived from glial cells which include astrocytes, oligodendrocytes, and microglia. Extracts from: Neuropathology Rapid Review_Q&A Arrangement
Marc Imhotep Cray MD
229
Cell Types in CNS: Neurons and Glia The CNS is composed of two predominant cell types, neurons and glia, each of which has many morphologically and functionally diverse subclasses Glial cells outnumber neurons and contain many neurotransmitter (NT) receptors and transporters There are 4 main types of CNS glial cells: 1. Astrocytes 2. Oligodendrocytes 3. Microglia 4. Ependymal cells Marc Imhotep Cray MD
Wecker L, et al. Brody’s human pharmacology : molecular to clinical 5th ed. Philadelphia, PA: Mosby, 2010.
NB: Read First AID for the USMLE Step 1, 2020. Neurology /Anatomy and Physiology, Pgs. 493-495. 230
Types of Neuroglia
https://qbi.uq.edu.au/brain-basics/brain/brain-physiology/types-glia Marc Imhotep Cray MD
231
Glia Cells Function, Astrocytes Astrocytes physically separate neurons and multineuronal pathways, assist in repairing nerve injury, and modulate metabolic and ionic microenvironment Astrocytes express ion channels and neurotransmitter (NT) transport proteins and play an active role in modulating synapse function express a range of receptors and transporters, and release a wide variety of mediators, including Glutamate, D-serine, ATP, Lipid mediators, Growth Factors etc… Marc Imhotep Cray MD
232
Glia Cells Function, Oligodendrocytes Oligodendrocytes form myelin sheath (in CNS) around axons and play a critical role in maintaining transmission down axons Polymorphisms (e.g. SNP*) in genes encoding several myelin proteins have been identified in tissues from patients with both schizophrenia and bipolar disorder and may contribute to underlying etiology of these disorders Note:*Developments in DNA sequencing now make it easy to look for allelic versions of a gene by sequencing samples of the gene taken from different members of a population (or from a heterozygous individual). Alleles whose sequence reveals only a single changed nucleotide are called single nucleotide polymorphisms or SNPs. SNPs most commonly refer to single-base differences in DNA among individuals useful for finding genes that contribute to disease. Marc Imhotep Cray MD
233
Glia Cells Function, Microglia Microglia proliferate after injury or degeneration (gliosis), move to sites of injury, and transform into large macrophages (phagocytes) to remove cellular debris These antigen presenting cells (APC) w innate immune Fx also appear to play a role in endocrine development
Glia Cells Function, Ependymal cells Ciliated simple columnar glial cells line ventricles and central canal of spinal cord Apical surfaces are covered in cilia (which circulate CSF) and microvilli (which help w CSF absorption)
Specialized ependymal cells (choroid plexus) produce CSF
Marc Imhotep Cray MD
234
Cell Types in CNS: Neurons Structural components of nerve cells.
Neurons are major cells involved in
intercellular communication b/c of their ability to conduct impulses and transmit information They are structurally different from other cells, w four distinct features: Dendrites A perikaryon (cell body or soma) An axon A nerve (or axon) terminal Wecker L, et al. Brody’s human pharmacology : molecular to clinical 5th ed. Philadelphia, PA: Mosby, 2010. Marc Imhotep Cray MD
235
Vignette 20 A 5-year-old boy presents to your office complaining of diminished vision and eye pain in his left eye. Physical examination reveals strabismus and a cat’s eye pupillary reflex. Funduscopic examination suggests an intraocular mass. When you hear that the boy’s father has had eye neoplasms, you order an MRI of the orbits and also refer this patient to both an oncologist and a medical genetics clinic. You fear that this boy may develop other cancers later in life. Wat is the Diagnosis?
Marc Imhotep Cray MD
236
Retinoblastoma Etiology: Caused by homozygous deletion in both alleles of RB gene, a tumor-suppressor gene located on chr 13, which results in a tumor arising from neuroepithelial cells in retina; most common intraocular neoplasm of childhood Can be either familial or sporadic o Familial form is transmitted as autosomal dominant trait even though homozygosity is necessary for disease o Over 90% of heterozygous carriers end up developing disease
Pathology Retina: Round cells w hyperchromatic nuclei & little cytoplasm arranged in Flexner Wintersteiner rosettes (cuboidal cells positioned around central lumen) May metastasize to brain, spinal cord, bone, or lymph nodes DDx of Flexner rosettes: Neuroblastomas have Homer-Wright rosettes tumor cells surrounding neuropil Ependymal rosettes, found in Ependymoma tumor cells w an empty lumen Marc Imhotep Cray MD
237
Retinoblastoma (2) Clinical Manifestations: Classically occurs in young children (familial form) who present w diminished visual acuity, eye pain, strabismus, intraocular mass on funduscopic exam, and white cat’s eye pupillary reflex Pts w familial disease develop bilateral retinoblastoma and are at an increased risk for developing other cancers (eg, osteosarcoma) Treatment and Prognosis: Surgery (removal of tumor or eye ) & radiation Tumor is fatal once it has spread beyond eye OF NOTE: Prototype of Knudson two-hit hypothesis: Two mutations are required for disease One deletion is either inherited (familial) or occurs sporadically Second mutation results from a sporadic mutation in both familial and sporadic cases Marc Imhotep Cray MD
238
Retinoblastoma, funduscopy
Klatt EC. Robbins and Cotran Atlas of Pathology, 3rd Ed. Saunders, 2015.
Marc Imhotep Cray MD
239
Retinoblastoma, gross
Klatt EC. Robbins and Cotran Atlas of Pathology, 3rd Ed. Saunders, 2015.
Marc Imhotep Cray MD
240
Retinoblastoma, microscopic
Klatt EC. Robbins and Cotran Atlas of Pathology, 3rd Ed. Saunders, 2015.
Marc Imhotep Cray MD
241
Vignette 21 A 59-year-old man presents to your office complaining of severe headaches for the last week. He tells you that his headaches are often associated with projectile vomiting. Physical examination reveals bilateral papilledema and a CT scan of the head demonstrates an irregular mass in the left cerebral hemisphere. When a CT-guided brain biopsy demonstrates pseudopalisading malignant cells around areas of necrosis, you realize that this patient’s prognosis is very poor. What is the Diagnosis?
Marc Imhotep Cray MD
242
Glioblastoma Multiforme Etiology and Epidemiology: Assoc. w genetic mutations in several oncogenes and tumor-suppressor genes, including TP53 (p53), RB gene (Learn them, First AID 2020, Pg. 224.) Most common primary brain neoplasm Occurs most frequently betw. ages 40 and 60 Pathology (GBM is a Grade IV astrocytoma) Gross: Variable, noncircumscribed lesion found in cerebral hemisphere Microscopic: High-grade astrocytoma (arises from astrocytes); central areas of necrosis and hemorrhage surrounded by multiple tumor cells, arranged in a pseudopalisading fashion; high degree of anaplasia Marc Imhotep Cray MD
243
Glioblastoma Multiforme (2) Clinical Manifestations: Seizures; headaches; nausea and vomiting; other signs of increased intracranial pressure Imaging: Mass effect, cerebral edema (discussed in Lectures. to follow.) Treatment and Prognosis: Surgical resection w chemotherapy and radiation Px is very poor w most pts. dying within a year of Dx Note: Low-grade astrocytomas, especially pilocytic astrocytomas, are benign, slow-growing tumors occurring in cerebellum of children On histology, Rosenthal fibers (eosinophilic, corkscrew fibers) are present Marc Imhotep Cray MD
244
Glioblastoma, gross
Klatt EC. Robbins and Cotran Atlas of Pathology, 3rd Ed. Saunders, 2015.
Marc Imhotep Cray MD
245
Glioblastoma, MRI
Klatt EC. Robbins and Cotran Atlas of Pathology, 3rd Ed. Saunders, 2015.
Marc Imhotep Cray MD
246
Glioblastoma, microscopic
Klatt EC. Robbins and Cotran Atlas of Pathology, 3rd Ed. Saunders, 2015.
Marc Imhotep Cray MD
247
Vignette 22 A 42-year-old woman presents to your office complaining of headaches and vomiting over the past 4 months. Upon further questioning, you learn that she also feels as if she trips more than usual when she is walking and she has recently had more trouble remembering things. A physical examination reveals bilateral papilledema and reduced strength and hyperreflexia in both of her legs. A CT scan reveals a parasagittal mass compressing the brain and a CTguided brain biopsy demonstrates a whorled pattern of tumor cells with psammoma bodies. You tell the patient that she will most likely need surgery to remove the tumor. What is the Diagnosis?
Marc Imhotep Cray MD
248
Meningioma Etiology and Epidemiology: Benign, slow-growing tumor arising from meningothelial cells of arachnoid; therefore external to brain Multiple meningiomas can be present in pts w neurofibromatosis type 2 Occurs most often in women after age 30 Pathology: Gross: Usually round encapsulated mass w dural base; usually occurs in convexities of cerebral hemispheres or parasagittal region; usually does not infiltrate brain Microscopic: Whorled pattern of tightly packed tumor cells; psammoma bodies (laminated calcifications) Marc Imhotep Cray MD
249
Meningioma (2) Clinical Manifestations: Sx are assoc. w compression of underlying brain including seizures, headaches, N/V and other signs of increased ICP Treatment and Prognosis: Surgical removal of tumor Prognosis is good Of Note: Meningiomas are second most common primary brain tumors
Marc Imhotep Cray MD
250
Meningioma, MRI
Klatt EC. Robbins and Cotran Atlas of Pathology, 3rd Ed. Saunders, 2015. Marc Imhotep Cray MD
251
Meningioma, gross
Klatt EC. Robbins and Cotran Atlas of Pathology, 3rd Ed. Saunders, 2015.
Marc Imhotep Cray MD
252
Meningioma, microscopic
Klatt EC. Robbins and Cotran Atlas of Pathology, 3rd Ed. Saunders, 2015.
Marc Imhotep Cray MD
253
Vignette 23 A 49-year-old man presents to the emergency department with a seizure. When he has been stabilized, he tells you that he has had several severe headaches over the past couple of months. A CT scan of his head reveals a large mass in the frontal lobe of his brain that demonstrates areas of calcification. You admit him to the neurology service and you suspect that he will need surgery to treat his condition. What is the Diagnosis?
Marc Imhotep Cray MD
254
Oligodendroglioma Etiology and Epidemiology: Relatively rare benign tumor derived from oligodendrocytes; Commonly affects middle-aged people Pathology: Gross: Circumscribed, slow-growing gray mass often w cysts; usually occurs in white matter of cerebral hemispheres (especially frontal lobe) Microscopic: Sheets of uniform cells with round nuclei with clear cytoplasm (fried egg appearance); often calcification is present; increased vascularity (“Chicken-wire” capillary pattern)
Clinical Manifestations: Seizures; headaches; papilledema; other signs of increased intracranial pressure (ICP) Imaging: Calcification of tumor is detected on CT scan
Treatment and Prognosis: Surgical resection, followed by radiotherapy and chemotherapy Average Marc Imhotep Cray MD
survival time is 5–10 years after diagnosis
255
Oligodendroglioma, MRI and microscopic
Klatt EC. Robbins and Cotran Atlas of Pathology, 3rd Ed. Saunders, 2015.
Marc Imhotep Cray MD
256
Vignette 24 A 58-year-old woman presents to your clinic complaining of hearing loss and a ringing in her left ear. The Weber and Rinne hearing tests help you to determine that the hearing loss is caused by a sensory disturbance and not a conduction deficit. You send the patient for an MRI of her head, which reveals a mass at the left cerebellopontine angle, impinging on cranial nerve VIII. You refer this patient to a neurosurgeon for a biopsy and likely removal of the mass. What is the Diagnosis?
Marc Imhotep Cray MD
257
Schwannoma (an acoustic neuroma) Etiology: Usually benign tumors arising from Schwann cells Bilateral acoustic schwannomas are assoc. w neurofibromatosis type 2 Pathology: Gross: Encapsulated masses, often with cystic areas; usually occur in cerebellopontine angle where it can compress cranial nerve VIII (acoustic schwannoma) Microscopic: Two growth patterns: (1) Antoni A: tightly packed elongated cells with palisading nuclei (2) Antoni B: loose arrangement of cells w microcysts
Marc Imhotep Cray MD
258
Schwannoma (2) Clinical Manifestations: Presents w Sx assoc. w compression of involved nerve (cranial nerve VIII compression leads to pts. presenting w ipsilateral hearing loss, tinnitus, and vertigo), seizures, headaches, N/V, and other signs of increased ICP Treatment and Prognosis: Surgical resection of tumor Prognosis is good Note: Pineal tumors usually occur in young men betw. ages of 10 and 40 Presents w Parinaud syndrome (paralysis of upward gaze caused by pre-tectal and superior colliculus damage, Obstructive hydrocephalus [owing to compression of aqueduct of Sylvius], and Endocrine abnormalities [owing to compression of hypothalamus]) Marc Imhotep Cray MD
259
Schwannoma, gross
Klatt EC. Robbins and Cotran Atlas of Pathology, 3rd Ed. Philadelphia: Saunders, 2015.
Marc Imhotep Cray MD
260
Schwannoma, microscopic
Klatt EC. Robbins and Cotran Atlas of Pathology, 3rd Ed. Philadelphia: Saunders, 2015.
Marc Imhotep Cray MD
261
Vignette 25 A 6-year-old boy presents to his pediatrician’s office complaining of frequent falls. Upon further questioning, you learn that the boy has also been suffering from nausea and vomiting, which is usually associated with headaches. During physical examination, you note that the boy has an ataxic gait and bilateral papilledema. You send the boy for a CT scan, which reveals a mass in the cerebellum and dilated third and lateral ventricles. You immediately refer the patient to a neurosurgeon. What is the Diagnosis?
Marc Imhotep Cray MD
262
Medulloblastoma Etiology and Epidemiology: Highly malignant tumor arising in cerebellum; assoc. w deletion on short arm of chr 17 (17p-) Occurs mostly in children and accounts for 20% of all brain tumors in children Pathology Gross: Gray, well-circumscribed tumor located at midline of cerebellum Microscopic: Hypercellular sheets of anaplastic cells, demonstrating many mitoses, scant cytoplasm, and hyperchromatic nuclei; cells are often arranged in a rosette or perivascular pseudorosette formation (called Homer Wright rosettes) Marc Imhotep Cray MD
263
Medulloblastoma (2) Clinical Manifestations: Unsteady gait; obstructive hydrocephalus (tumor may obstruct flow of CSF by compressing fourth ventricle); seizures, headaches; N/V; other signs of increased ICP Treatment and Prognosis: Surgery w radiation and ChemoTx With total excision and radiation, 5-year survival rate is 75%
Marc Imhotep Cray MD
264
Medulloblastoma, gross
Klatt EC. Robbins and Cotran Atlas of Pathology, 3rd Ed. Philadelphia: Saunders, 2015.
Marc Imhotep Cray MD
265
Medulloblastoma, MRI
Klatt EC. Robbins and Cotran Atlas of Pathology, 3rd Ed. Philadelphia: Saunders, 2015.
Marc Imhotep Cray MD
266
Medulloblastoma, microscopic
Klatt EC. Robbins and Cotran Atlas of Pathology, 3rd Ed. Philadelphia: Saunders, 2015.
Marc Imhotep Cray MD
267
Vignette 26 An 8-year-old girl presents to your clinic complaining of blurry vision. During physical examination, you note bilateral papilledema. A CT scan of the head demonstrates a mass extending from the floor of the fourth ventricle and dilated lateral and third ventricles. You suspect that a biopsy of the mass would demonstrate cells with blepharoplasts in a perivascular pseudorosette arrangement. What is the Diagnosis?
Marc Imhotep Cray MD
268
Ependymoma Etiology and Epidemiology: Tumor arising from ependyma of ventricular system Most commonly occur in children (usually in fourth ventricle), but can occur in spinal cord of adults Pathology: Gross: Solid, papillary masses extending from floor of fourth ventricle Microscopic: Uniform cells w round nuclei set in a fibrillary stroma and arranged in a perivascular pseudorosette formation; tumor cells often have blepharoplasts (rod near nucleus, which represents basal ciliary bodies)
Marc Imhotep Cray MD
269
Ependymoma (2) Clinical Manifestations: Obstructive hydrocephalus (tumor may obstruct flow of CSF through compression of fourth ventricle); seizures; headaches; N/V; other signs of increased ICP Treatment and Prognosis: Surgical excision (difficult owing to proximity of brainstem nuclei) Px is poor w average survival time of 4 years
Marc Imhotep Cray MD
270
Ependymoma, CT image
Klatt EC. Robbins and Cotran Atlas of Pathology, 3rd Ed. Philadelphia: Saunders, 2015.
Marc Imhotep Cray MD
271
Ependymoma, gross
Klatt EC. Robbins and Cotran Atlas of Pathology, 3rd Ed. Philadelphia: Saunders, 2015.
Marc Imhotep Cray MD
272
Ependymoma, microscopic
Klatt EC. Robbins and Cotran Atlas of Pathology, 3rd Ed. Philadelphia: Saunders, 2015.
Marc Imhotep Cray MD
273
Vignette 27 A 2-year-old boy is brought to your office after his parents noticed a large abdominal mass while dressing him. They also remark that he seems to have lost weight over the past month. During physical examination, you note ecchymoses over his trunk, a large palpable abdominal mass, and a blood pressure of 160/100. Laboratory tests reveal an elevation of urinary VMA and catecholamines and a CT scan of the abdomen demonstrates a large mass arising from the right adrenal gland. When a biopsy of the mass reveals HomerWright pseudorosettes, you suspect that the amplification of a specific oncogene may be responsible for his condition. What is the Diagnosis?
Marc Imhotep Cray MD
274
Neuroblastoma Etiology and Epidemiology: Tumor arising from neural precursor cells; assoc. w amplification of N-myc oncogene and deletions in short arm of chr 1 (1p-) Most commonly seen in young children, but does rarely occur in adults Pathology: Gross: Classically arises in adrenal medulla but can arise in sympathetic chain, pelvis, neck, or brain; variable in size; may be circumscribed; may show cyst formation or necrosis Microscopic: Sheets of small cells w dark nuclei and scant cytoplasm often arranged in Homer Wright pseudorosettes; neurosecretory granules containing catecholamines Marc Imhotep Cray MD
275
Neuroblastoma (2) Clinical Manifestations: Classic presentation is young child (< 2 years old) w large abdominal mass, hypertension, and weight loss Other Sx include ecchymosis and proptosis (protrusion of eyes) Older children may present w symptoms of metastases to bone, liver, or lungs (manifesting as bone pain, respiratory, or GI symptoms) Lab findings: Increased 24-hour urinary VMA and metanephrine levels, increased plasma and urinary catecholamine levels
Treatment and Prognosis: Surgical resection with chemotherapy Px is variable; younger age of pt. and lower stage of cancer at Dx has a better prognosis
Marc Imhotep Cray MD
276
Adrenal neuroblastoma, gross
Klatt EC. Robbins and Cotran Atlas of Pathology, 3rd Ed. Philadelphia: Saunders, 2015.
Marc Imhotep Cray MD
277
Adrenal neuroblastoma, microscopic
Klatt EC. Robbins and Cotran Atlas of Pathology, 3rd Ed. Philadelphia: Saunders, 2015.
Marc Imhotep Cray MD
278
Vignette 28 A 14-year-old girl presents to your clinic complaining of multiple nodules on her skin. She tells you that her mother suffers from a similar condition. Upon further examination, you find multiple coffee-colored macules on her torso and limbs and pigmented nodules on her irises. You suspect that her condition is caused by an autosomal dominant genetic disorder and you refer her to a medical genetics clinic. What is the Diagnosis?
Marc Imhotep Cray MD
279
Neurofibromatosis Type 1 Etiology: Autosomal dominant disorder that is caused by a mutation in neurofibromatosis type 1 (NF1) gene a tumor suppressor gene located on chr 17 Pathology: Neurofibroma: Unencapsulated, well-circumscribed masses of spindle cells, which occur in dermis (cutaneous), in peripheral nerve (solitary), or in a large nerve trunk (plexiform); cutaneous neurofibromas are visible as skin nodules and may cause hyperpigmentation of overlying skin
Marc Imhotep Cray MD
280
Neurofibromatosis Type 1 (2) Clinical Manifestations: Neurofibromas may cause neurologic symptoms (eg, gliomas of optic nerve may lead to visual disturbances) Lisch nodules: Pigmented nodules of iris Café-au-lait spots: Cutaneous pigmented macules Treatment: Surgery to remove neurofibromas if disfiguring or causing neurologic abnormalities Neurofibromatosis type 2 is an autosomal dominant disorder that is caused by a mutation in neurofibromatosis type 2 (NF2) gene, located on chr 22 It is rarer than neurofibromatosis type 1 and presents w bilateral acoustic schwannomas, multiple meningioma, and other neoplasms Marc Imhotep Cray MD
281
Neurofibromatosis, gross
Klatt EC. Robbins and Cotran Atlas of Pathology, 3rd Ed. Philadelphia: Saunders, 2015.
Marc Imhotep Cray MD
282
Neurofibroma, microscopic
Klatt EC. Robbins and Cotran Atlas of Pathology, 3rd Ed. Philadelphia: Saunders, 2015.
Marc Imhotep Cray MD
283
Increased Intracranial Pressure (ICP) Raised ICP may occur secondary to intracranial hemorrhage (usually acute onset) or as a result of a space-occupying lesion such as a neoplasm (usually gradual onset) Early effects include cranial nerve compression (e.g. third nerve compression leading to pupillary dilatation) One of main dangers of increased ICP is it can cause brain ischemia by decreasing cerebral perfusion pressure (CPP) Later effects include herniation of brain tissue through an anatomical aperture (e.g. the foramen magnum), which when severe may lead to brainstem compression and death Marc Imhotep Cray MD
284
Pathophysiology of Increase ICP CPP, pressure of blood flowing to brain, is normally fairly constant due to autoregulation, but for abnormal MAP or abnormal ICP, cerebral perfusion pressure is calculated by subtracting intracranial pressure from mean arterial pressure: CPP = MAP â&#x2C6;&#x2019; ICP . One of main dangers of increased ICP is that it can cause ischemia by decreasing CPP. Once ICP approaches level mean systemic pressure, cerebral perfusion falls. The body's response to a fall in CPP is to raise systemic blood pressure and dilate cerebral blood vessels. This results in increased cerebral blood volume, which increases ICP, lowering CPP further and causing a vicious cycle.
Marc Imhotep Cray MD
285
Lobar hemorrhage, gross
Klatt EC. Robbins and Cotran Atlas of Pathology, 3rd Ed. Philadelphia: Saunders, 2015.
Marc Imhotep Cray MD
286
Cerebral edema, gross
Klatt EC. Robbins and Cotran Atlas of Pathology, 3rd Ed. Philadelphia: Saunders, 2015.
Marc Imhotep Cray MD
287
Cerebral edema, gross
Klatt EC. Robbins and Cotran Atlas of Pathology, 3rd Ed. Philadelphia: Saunders, 2015.
Marc Imhotep Cray MD
288
Cerebral edema, MRI
Klatt EC. Robbins and Cotran Atlas of Pathology, 3rd Ed. Philadelphia: Saunders, 2015.
Marc Imhotep Cray MD
289
Brain Herniation Syndromes|Types 1. Uncal • Blown pupil (3rd nerve palsy, compression from uncus into tentorium cerebelli • Hemiparesis/ Babinski (75% contralateral) 2. Central Transtentorial • Pinpoint → nonresponsive midpoint pupils • Motor weakness → increased tone → flexor, then extensor posturing 3. Cerebellotonsilar (posterior fossa pressure) • Pinpoint pupils • Sudden respiratory and cardiovascular collapse • Flaccid quadriplegia 4. Upward Transtentorial • Pinpoint pupils • Downward conjugate gaze 5. Subfalcine herniation (cerebral mass lesion causes medial surface of affected hemisphere to push against rigid falx cerebri, then herniate underneath) • Cingulate gyrus most commonly affected • Often clinically silent The Global Emergency Medicine Wiki https://wikem.org/wiki/Brain_herniation_syndromes Marc Imhotep Cray MD
290
Brain Herniation Syndromes Illustrated
Marc Imhotep Cray MD
The Global Emergency Medicine Wiki https://wikem.org/wiki/Brain_herniation_syndromes
291
Le T and Bhushan V. First Aid for the USMLE Step 1 2020. M-H, 2020, Pg. 529.
Marc Imhotep Cray MD
292
UWorld Question Check Up A 55-year-old right-handed woman complained of acute onset headache and difficulty with vision. She quickly becomes unconscious en route to the hospital. She has a past medical history of hypertension. Her medications include aspirin and lisinopril at home. Her blood pressure is 150/90 mm Hg and pulse is 90/min and regular. A stat head CT scan without contrast demonstrates an acute hemorrhage in the left temporal lobe and compression of the anterior medial temporal lobe against the free margin of the tentorium cerebelli. Which of the following cranial nerves is most likely to be compromised in this patient? A. Optic B. Oculomotor C. Trochlear D. Trigeminal E. Abducens F. Facial G. Vagus
Marc Imhotep Cray MD
293
Answer, B. Oculomotor Educational objective: Major herniations of the brain Transtentorial (i.e., uncal) herniation is a complication of an ipsilateral mass lesion, such as a hemorrhage or brain tumor. The first sign of uncal herniation is a fixed and dilated pupil on the side of the lesion. lpsilateral paralysis of oculomotor muscles, contralateral or ipsilateral hemiparesis, and contralateral homonymous hemianopsia with macular sparing may also occur.
Source: UWorld Step 1, 2015. Neurology/Pathology Q#40 Marc Imhotep Cray MD
294
Herniation, gross
Klatt EC. Robbins and Cotran Atlas of Pathology, 3rd Ed. Philadelphia: Saunders, 2015.
Marc Imhotep Cray MD
295
Herniation with Duret hemorrhages, gross
Klatt EC. Robbins and Cotran Atlas of Pathology, 3rd Ed. Philadelphia: Saunders, 2015.
Marc Imhotep Cray MD
296
Cerebellar tonsillar herniation, gross
Klatt EC. Robbins and Cotran Atlas of Pathology, 3rd Ed. Philadelphia: Saunders, 2015.
Marc Imhotep Cray MD
297
Brain herniation - causes, symptoms, diagnosis, treatment, pathology_Osmosis
Marc Imhotep Cray MD
298
THE END
See next slide for sources and links to tools for further study. Marc Imhotep Cray MD
299
Sources and tools for further study Presentation Sources: Baron SJ; Lee, CI. Lange Pathology Flash Cards, 2nd Ed. Mc Graw-Hill, 2009. Brown TA, Shah SJ. USMLE Step 1 Secrets 3rd ed. Sanders, 2013. Klatt EC. Robbins and Cotran Atlas of Pathology 3rd Ed. Saunders, 2015. Kumar V and Abbas AK. Robbins and Cotran Pathologic Basis of Disease, 9th ed. Saunders, 2015. Le, T; Bhushan, V. (Eds.) First AID for the USMLE Step 1 2020. Mc Graw-Hill, 2020. Le, T; Krause, K (Eds.) First AID for Basic Sciences: Organ Systems. Mc GrawHill, 2009. Textbooks: Kumar V and Abbas AK. Robbins and Cotran Pathologic Basis of Disease 8th ed. Saunders, 2014. Rubin R and Strayer DS Eds. Baltimore: Lippincott Williams & Wilkins, 2012. Companion Study Tools: Nervous System Pathology Notes Neuropathology Rapid Review Notes_Q&A Arrangement Nervous System Pathology Practice Questions Marc Imhotep Cray MD
300